You are on page 1of 80

TABLE OF CONTENTS CON LAW I

I. FRAMING AND INTERPRETATION OF CONSTITUTION, AND


THE POWERS OF THE COURTS............................................................................2
a) The Constitution......................................................................................................2
b) Approaches to Interpreting the Constitution............................................................2
c) The Role of the Courts in the Constitutional Framework........................................2
Marbury v. Madison (1803) pg. 22..............................................................................2
d) Uses of Judicial Review...........................................................................................2
e) Limitations on the Powers of the Federal Courts....................................................2
f) Advisory Opinions, Standing, and Political Question Doctrine..............................2
Standing.......................................................................................................................2
Political Question Doctrine..........................................................................................2
Ripeness and Mootness................................................................................................2
II. THE SEPARATION OF POWERS................................................................2
Executive Authority:........................................................................................................2
Executive Privilege, Impeachment, and Immunity......................................................2
Legislative Authority...................................................................................................2
III. THE POWERS OF CONGRESS....................................................................2
Commerce Clause............................................................................................................2
Pre-New Deal...............................................................................................................2
New Deal.....................................................................................................................2
The Revival of Limitations on Congresss Commerce Clause Powers...........................2
The 10th Amendment as an External Limit on the Scope of Commerce Clause............2
Eleventh Amendment as a Barrier to Enforcement of Commerce Clause.......................2
Others Powers of Congress..............................................................................................2
The Treaty and War Powers.........................................................................................2
The Spending Powers..................................................................................................2
The Power to Enforce the Reconstruction Amendments.............................................2
IV. Constitutional Limits on State Powers (Dormant Commerce Clause)..2
The Prohibition of Facial Discrimination against IC.......................................................2
Exceptions to the Prohibition of Facial Discrimination against IC.................................2
Facially Neutral Statutes..................................................................................................2
Preemption.......................................................................................................................2
The Contracts and Takings Clauses.................................................................................2
Lochner Era.....................................................................................................................2

US v. Curtiss-Wrightpg. 2 Gibbons v. Ogdenpg. 27


McCulloch v. Marylandpg. 3 Martin v. Hunters Lesseepg. 8
Marbury v. Madisonpg.6 Wickard v. Filburnpg. 32
Schecter Poultrypg. 30 US v. Lopezpg. 34
Ex Parte McCardlepg. 11 NY v. USpg. 37
Allen v. Wrightpg.12 Seminole v. FLpg. 41
Baker v. Carrpg. 15 US v. Butlerpg. 46
Youngstownpg. 18 Philly v. NYpg. 53
US v. Nixonpg. 20 Blaisdellpg. 68
INS v. Chadhupg. 21 Lochner v. NYpg. 70

CON LAW I Outline Page 1


I. FRAMING AND INTERPRETATION OF CONSTITUTION,
AND THE POWERS OF THE COURTS
a) The Constitution
INTRODUCTION
This course is about the courts protecting individual rights against the majority. We will
also deal with the structure of government (i.e. separation of powers and federalism).
Hamilton and the Bill of Rights (BOR)
Hamilton argues that the Bill of Rights was not included originally with the articles of
confederation (AOC) because it would have been limiting in the sense that; what about
all of the other things. Under the AOC the government was very weak.
Hamiltons arguments against a (BOR):
1. You list some and dont list others
2. If you put a restraint on the government you assume that without the restraint the
power exists. Maybe if we limit the powers of the government there is no need for
a BOR.
Hamilton argued that declaring rights is unnecessary because enumerating powers
provide enough protection.

US v. Curtiss-Wright Corp. (1936) pg. 403


F: Charge against company for conspiring to sell arms to Bolivia in violation of the Joint
Resolution of Congress. The lower court ruled that the Act (presidential proclamation)
was unconstitutional and that the President could not prohibit the sale of arms.
ROL: Presidential proclamations pertaining to foreign affairs are valid and enforceable.
R: Because federal power over external affairs in origin and character is different from
that over internal affairs. The federal government has always had exclusive authority in
the area of foreign affairs.
Class notes:
There was no question about whether Congress could pass legislation outlawing arms
sales, however the issue here related to the Presidential powers and the delegation of
powers from the Congress to the President.
Does the federal government get its powers only from the Constitution? According to this
case, when it comes to domestic powers they are limited to the enumerated powers from
the constitution, and foreign powers are not only from the C.

Do these issues really protect individual liberties? Is it so good to have a government


with limited powers (enumerated)? Limiting governments entrenches existing interests.
Perhaps we need to re-think the idea that having a powerful government is a negative.

Pp 1-22 The Federalist (the framers) and Anti-Federalist (AF) Argument


While the C is democratic, it also limits the influence of democracy and majorities, to
make the govt less subject to popular controls. (When it comes to constitutional
interpretation we consider: Text Intent Structure or Function).

CON LAW I Outline Page 2


Hypo about abortion clinic from syllabus:
The AF view:
The AF would prefer a town meeting based on the idea that it is more democratic that
everyone has a say and that they would be involved in the political process and debate the
issues. Although the AFs realized that not everything could be decided by a town
meeting, the ideal model makes a difference. AFs would probably advocate a referendum
in the same sense that they like the town meeting model. The AFs are for this because of
participation and the idea that you are more likely to reach the right decision through
public spirit. This view is similar to the jury approach of requiring anonynimity, which
promotes debate.
The Federalist view:
The town meeting is a bad thing because minority rights would be overlooked (factions)
and because the debate really will not be so enlightening and people will just vote based
upon their preset views. The Fed. view does not want to see the majority constantly
trample over the minority. Madisons view is that the larger the republic the better it is
because the representatives will be the top and the cream of the crop (elite). Madison did
not want the representatives to be too directly restrained.

Anti-Federalists believed in decentralization. Traditional republican theory. Opposed a


dramatic expansion in the powers of national government. Did not like the idea of
government by representatives as opposed to decisions made by the people themselves.
Anti-feds desired to avoid extreme disparities in wealth, education, and power. Anti-feds
thus opposed the proposed Constitution on the grounds that it was against traditional
republicanism, removed the people from the political process, and created a powerful and
remote national government.
The articles of confederation did not give enough power to the national government. The
confederation was more like many states acting under treaties than it was one entity.

The Sources of Judicial Decisions


The following case is about how we go about interpreting the constitution. Madison v.
Marbury will deal with the question of whose role it is to interpret the C (ultimately
established judicial review).

b) Approaches to Interpreting the Constitution

McCulloch v. Maryland (1819) pg. 55


(Congress is in a better position to uphold the principles of Federalism.)
F: Marshall - Action against D (cashier) in a bank for failure to pay a state tax(expensive
tax) assessed against the bank. The court below held for the P. The cashier of the US bank
issued bank notes in violation of a MD statute providing that no bank, without state
authority, could issue bank notes except on stamped paper issued by the state.
I: Whether: 1. Congress has the power to incorporate a bank.
2. The state of MD may tax the bank without violating the constitution.
H: Yes and no, reversed.
R: Because congress has the discretion and power to choose and enact the means to
perform the duties imposed upon it from the Necessary and Proper Clause. The C does

CON LAW I Outline Page 3


not exclude implied powers. Not everything must be laid out which would be
impossible. (MD says necessary is limiting while the D argues it means useful and
essential).
The C derives its power from the people, and not from the states. MD was wrong in its
contention that the powers of federal govt are delegated by the states. The power to tax is
the power to destroy. The states have no power to impede, burden, or control the
operations of constitutional laws enacted by congress. The MD statute is unconstitutional
and void.
ROL: (1) Certain federal powers giving Congress discretion and power to choose and
enact the means to perform the duties imposed upon it are to be implied from the
Necessary and Proper Clause. (2) The Federal constitution and the laws made pursuant to
it are supreme and control the Constitutions and laws of the states.
Class notes:
The issue is whether Congress has the power to create a National Bank. And then if they
did have this power, could MD tax this federal bank.
Tax was very high. It would have an adverse affect/interfere with the banks ability to
conduct business.
Constitutional Construction:
We cannot take the fact that something is absent in the text as definitive because
otherwise you have a legal code and not a C. The C cannot be expected to include all
possibilities. Even though an express prohibition in the C against setting up a bank would
disallow it, it does not follow that its absence in the C does not. You would not expect to
find everything explicitly in the C.
There are 2 approaches:
1. Broad framework with brevity flexible, more accessible, democratic and
understandable due to its brevity.
2. Detailed would have to cover everything and would be static.

[The 11th amendment prohibits a citizen of FL from suing the state of FL in a federal
court (this is pretty much ignored now).]

One might argue that since the C spelled out the right to establish post offices (Article 1,
section 8, clause 7) that it says something about the absence of a bank (that FRAMERS
DID NOT INTEND FOR A NAT. BANK).
On the other hand, opposition may say it was intentionally left out because it is so
obvious that they have a right to establish a bank.
One might argue that there is an implied power to make a national bank because the C
does not mention it explicitly but it does mention powers to:
Collect taxes, pay debt, go to warand you may conclude that it implies a power to
create a national bank as a means and that you are not adding any new powers to
Congress. You are just creating a means to implement the powers that Congress already
has.
Clause 5 addresses the power to coin money so one could not say that the framers did not
forget about monetary issues. Then clause 6 says that Congress can punish against
counterfeiting which you may argue is superfluous because if you are given the power to
do something you should also be able to regulate it. Regarding the post office you may

CON LAW I Outline Page 4


argue that since punishing against counterfeiting was spelled out you may conclude that
Congress cannot punish for crimes against the post office.

Back to this case specifically:


MDs argument in terms of their vision of the US: They view the states more like
a UN and that it should not be easy to infer a power on the federal government.
They want to make it difficult and be limiting when giving a power to the federal
government. The MD view of the nation is a collection of sovereign entities.

Marshalls argument is that the power comes from the people and not the states.
His view of the nature of the union is different than that of MD. Marshall says
that they could not have just had one large national convention because that would
be impractical. Marshall (as a federalist) wants to see a strong national
government. Marshall is saying that Congress can setup the bank even without the
Necessary and Proper Clause.

Necessary and Proper Clause (N&P) According to Marshall, Necessary should be


construed as useful, desirable or helpful, but that it does not have to be so strict to say that
it has to be indispensable and the only way. A government would not work if it were
construed so strictly to mean indispensable. Also if you look at the context of where the
N&P clause is, it is in a section that gives powers to Congress. If, for example, it were in
section 9, which restricts congressional powers, it would be easier to argue the strict
construction of the N&P clause. Article 1, Section 10, clause 2, which limits the
powers of the states says absolutely necessary so you can argue that necessary
alone cannot therefore be construed as absolutely necessary.
The 10th amendment in talking about delegated powers to Congress omits the
word, expressly while the Articles of Confederation mentions the phrase. This
implies that the powers are not limited to things expressly laid out.

Dudgeon v. UK (1982) pg. 81 supp. European Court of Human Rights


Man in N. Ireland sues because of the laws criminalizing homosexual activity. The law is
overturned as a breach of Article 8 of the Convention. P argued he suffered fear, suffering
and stress because of the laws. No evidence that such activity is injurious to moral
standards.
Class notes:
This case has to do with the right of privacy. This case also deals with how we interpret
necessary. Under Marshalls interpretation of necessary people would be entitled to
less privacy. This opinion construes necessary to be less flexible to mean a pressing
social need in this context, and that the means must be proportionate to the end.

How do we reconcile the 2 constructions of the same word in these 2 cases? One may
distinguish that one case involves state sovereignty where the other deals with individual
rights where we should construe necessary narrowly to protect individual rights.

Issue of whether the states can tax the national bank:

CON LAW I Outline Page 5


MDs argument: Section 10 deals with limitations on the powers of the states and deals
with monetary issues, yet it does not say explicitly that the states cannot tax the national
government.
Continuation of McCulloch v. Maryland. Can Maryland tax a federal bank?
MD argues that the power to tax is fundamental and that the C does not mention that the
state cannot impose a tax.
MD argument: If you restrict MD from taxing, then you will be able to limit their powers
in a far reaching amount of areas. No constituional text expressley banning the taxation
of a federal bank
Marshalls arguments:
Marshall distinguishes that there is no worry about MD taxing its own citizens unjustly
because they have recourse through voting and participating in government. This is not
the case where MD taxes the federal government. This is a structural argument in terms
of what works. There was nothing in the C about taxing the bank (but they argue its
implied in the having of separation of powers this ties into US v. Nixon. To have
checks and balances, you need effectiveness of the branch, and so sometime syou have to
argue that the power to tax was implied). Marshall has no problem here having politicians
make political decisions, as long as they will be accountable to those who are impacted
by the decisions. Marshall is against the idea that MD can tax people who do not have a
say in the matter. Representation Reinforcement there is an idea that the court should
intervene where the electoral process does not work very well. This also comes into play
with the Dormant Commerce Clause. The main point of this case has to do with
constitutional interpretation. We must think about the starting perspective: as the people,
or as MD as a loose collection. This predisposition is very important. You can get into the
question of the intent of the framers.

1) Maryland as states, only gave up a few powers; they are sovereign; does not
make sense that states would have agreed to any limitations
2) Marshall what is the basis of the limitation? Article 6 Section 2 must follow
the federal law, laws are based on interpretation of C

Flaw in system tax would affect other states, and other states would not have a say in
Maryland affairs

Any middle ground? Limits on taxing by federal statute? Article 6 Supremacy Clause

Schlenby Thoughts: Other forms/models of government are possible, many different


ways to interpret documents. Countries such as South Africa have much more detail in
their Constituiton.

c) The Role of the Courts in the Constitutional Framework


Marbury v. Madison (1803) pg. 22

(Political Question)
F: At the end of President Adams term, he appointed Marbury as justice of the peace and
the Senate approved the appointments. Then President Jefferson took over and refused to

CON LAW I Outline Page 6


deliver the commissions. Madison was Jeffersons Sec. of state who refused to deliver the
commission.
I: Whether the C gives the SC the authority to review acts of Congress and declare them
void if they are repugnant to the C.
H: Yes
R: Article VI, section 2 of the C gives the SC the power to review acts of Congress. The
government of the US is a government of laws, not of men. When the President or his
officers act, they are not subject to examination by the court, however where the officers
are given by law specific duties on which individual rights depend, the law must provide
a remedy.
The Judiciary Act of 1789 authorized the courts to issue writs if mandamus*. However
according to the C the SC was a court of appellate jurisdiction with certain exceptions
where it had original jurisdiction. For the SC to issue the writ of mandamus here would
give it original jurisdiction. This would be an unconstitutional exercise of original
jurisdiction beyond the power of the court. It is the duty of the judicial department to say
what the law is. Mandamus denied.
By signing the commission, a legal right was conferred upon Marbury to be justice of the
peace. A refusal to deliver he is entitled to a remedy under the law. The SC deals solely
with rights of the individuals. It does not inquire how the President performs duties in
which it has discretion. Questions that are political can never be made in this court.
However if an issue is not based upon political discretion, but rather it is based upon
principles of law or acts of Congress then the SC will decide.
ROL: The SC has the power to review acts of Congress and if they are found repugnant
to the Constitution, to declare them void.
Notes from reading: The actual holding here is that the SC does not have the power to
direct the President to deliver Marburys commission. The existence of judicial review,
was therefore established in a case in which the Court concluded that it had no power to
do anything to remedy official illegality. Instead the holding was that the SC has the
power to declare acts of Congress unconstitutional.

Class notes:
This case established judicial review. It is interesting that Marshall gave this opinion and
did not recuse himself since he was Sec. of State at the time of the event. If Marshall
concluded that the SC did not have jurisdiction, why did he first deal with the questions
regarding judicial review. Jefferson was not happy with this opinion. It stated that
Jefferson violated the law. Marshall did accomplish making a President of another party
look bad. To make a property analogy, Madison may argue that it is like a signed deed
that is not valid until it is delivered.
Is there jurisdiction under the Judiciary Act? Why did Marbury go straight to the SC with
this case? Looking at the text of the act on pg. 26, the Act says the SC shall have the
power to issue writs of mandamus to persons holding office under the authority of the
US. However, looking at the beginning of that sentence, the Act is talking about appellate
jurisdiction. Also, the prior section of the act deals with original jurisdiction, so you may
argue that taken in its entirety the writ of mandamus seems to deal with appellate
jurisdiction. It only sounds like original jurisdiction when the passage is read in isolation.

CON LAW I Outline Page 7


Marshall seems to look at that part in isolation that the Act does give the SC the power.
The next issue is whether that is consistent with the C (Article III)? We know that the
opinion concluded that the Judiciary Act is not consistent with Article III of the C. The
opinion says that Marbury was entitled to the commission, and that this was not a
political issue, however the remaining issue is whether the SC can deem a Congressional
Act to be unconstitutional.
Marshalls argument about why the Judiciary Act is inconsistent with Article III:
(Aside: there is not necessarily an obligation for Congress to establish lower courts at
all). Section 2 mentions Public Ministers which may or may not extend to a Sec. of
State. This follows and precedes Ambassadors and Consuls, which may suggest that this
does not extend to elected officials. (Why doesnt Congress want an Ambassador tried in
a state court? We are afraid that the court of one state will favor a side one way or another
just like a diversity of citizenship case, it is not that the state courts couldnt handle
international law). Schnably thinks the C is referring to foreign Ambassadors and Consuls
and similarly that public ministers refers to foreigners. The worry is about foreign
relations so the framers want these cases handled by federal courts.
The argument than shifts to say that this is under the judicial power because article III
says judicial power extends to all cases arising under the laws of the United States. So
Marshall argues that although there is judicial power here, the SC would only have
appellate jurisdiction. Marburys argument is that the enumerated instances where the SC
has original jurisdiction (Ambassadors and Consuls and parties of another state) is not an
exhaustive list (it does not say only in the following cases), and other instances can
be included. Marshalls holding is that Congress CANNOT add to when the SC has
original jurisdiction. Marshall argues that it would be a surplusage and superfluous to list
the couple of instances (Ambassadors etc.) and then say that other non-listed instances
will also have original jurisdiction.
There is also an appellate exceptions clause Marbury may use this clause to argue that
the SC should have jurisdiction. If too many exceptions are made though, one could
argue that everything would become original jurisdiction and the exceptions did away
with the rule. The Judiciary Act talks about original and not exclusive jurisdiction
(concurrent). Along those lines, Article III says that in cases regarding Ambassadors the
SC shall have original jurisdiction, however it does not say that it is exclusive. There is
exhaustive and then there is concurrent.

Is the Judiciary Act (section 13) consistent with Article III?


To interpret the constitution, we look at:
1. Text
2. Intent (framers historical record, change).
3. Structure - Function

A number of the people who passes the Judiciary Act in 1789 were in the First Congress
and were the framers this would indicate that adding power to the judiciary would be
consistent with the C because these same people framed the C only a couple of years
before the JA was written.
Does intent matter, and should intent even matter? The meaning changes or evolves over
time. Maybe looking at intent will constrain the judges.

CON LAW I Outline Page 8


Are the framers the people who wrote the C, or the people who adopted it? Whose intent
are we trying to figure out? Who are the framers?
Why is the C binding? We have power to change the C as we see fit through amendments.
Why should the SC have the final say on constitutional interpretation as opposed to
Congress?
1. They are best suited because they are insulated from politics (structural argument)
2. Text Article III section 2 The Judicial Power should extend to
3. What about the Supremacy Clause (Article VI)? Not a very strong argument.
4. 11th Amendment may be a textual argument.
What about the fact that it was the judiciary itself that made the decision that there should
be judicial review in the first place?

Martin v. Hunters Lessee (1816) pg. 45


(The Supreme Court shall have the power to review decisions of the States Supreme
Courts if they involve a federal question.) Article III Sec. 2 provides that the Supreme
Court appellate jurisdiction may be regulated and limited as congress shall provide.

F: Land dispute. The P (Hunter) claimed the land under a grant from VA that confiscated
lands owned by British subjects. D (Martin), a Brit, claimed that the attempted
confiscation was ineffective under anti-confiscation clauses of treaties between the US
and UK. Lower court held for Martin and the VA CA reversed for Hunter saying that the
states interest in the land had vested before the treaties were relevant and that the Act of
Compromise defeated Martins claim. The USSC reversed claiming that VA had not
perfected its title before the relevant treaties, and did not mention the Act of Compromise.
The USSC remanded the case to the VACA and they declined stating that section 25 of
the Judiciary Act was unconstitutional insofar as it extended the appellate jurisdiction of
the SC to the VA court.
ROL: The SC has appellate jurisdiction over issues of federal law in the state courts.
R: Because it is the case and not the court that gives the jurisdiction.
1. Article III says that judicial power shall extend to all cases arising under the C etc.
Article VI also makes this implication with the Supremacy Clause. The
constitution was designed to operate on the states.
2. The US courts can invalidate the state proceedings from the executive and
legislative authorities if it is contrary to the constitution. This is no more
dangerous than the same right over judicial tribunals. It cannot be said that USSC
review of state court decisions is overly intrusive on state power because it is
accepted that Congress can supersede state laws, and this is equally intrusive.
3. In certain cases there is an importance and a necessity of uniformity of decisions
throughout the whole US. The laws on Constitutional issues would vary from
state to state if the SC did not have appellate jurisdiction here.
4. Also, the C was designed for the common and equal benefit of all the people of
the US. The 25th section of the Judiciary Act does not limit the power of the
USSC.
Class notes:

CON LAW I Outline Page 9


This state establishes SC judicial review over the state courts. It would be hard to believe
that the SC could have judicial review over Congress and not over a state court. The
dispute was over the interpretation of the anti-confiscation treaty.

Is there jurisdiction under the statute for Martin (the Brit)?


If Martin had won, would this statute have given jurisdiction to the VA court for an
appeal? No

The opinion says that it cant be the case that we have no appellate jurisdiction over state
courts because per Article III judicial power extends to all cases. Virginia is arguing that
they would not have given up the finality of their authority of their state SC without
stating so in an explicit manner. As we review the Judiciary Act of 1789 (pg. 18
supplement) we see no such express provision.

[This is unclear and very confusing] The Act says that it only applies when you draw into
question the construction of a statute (treaty, const.) when you are against the one
claiming title under a statute (treaty, constitution). If a decision was in favor of the person
claiming title, such a case may never make it to the SC.
Arguments for federal courts to enforce the C:
1. Uniformity if it were done in the states you could have discrepant constructions
of the C.
2. To avoid bias and forum shopping.
If the case came out the other way saying that there was no appellate jurisdiction, you
would want to try and get these cases into federal courts and out of state courts. Virginia
took the position that no appellate review would protect the sovereignty of the state.
How much can you trust state judges to handle federal issues?

a) Sovereignty argument rejected: First, the Virginia Courts assertion that it was
sovereign was rejected on the grounds that the federal Constitution cut back upon the
states sovereignty in numerous respects. There was no reason to presume that the
state judiciaries were immune from this set of limitations.
b) Uniformity: Secondly, there is a need for uniformity in decisions throughout the
nation interpreting the Constitution. if there were no revising authority to control
these jarring and discordant judgments, and harmonize them into uniformity, the laws,
the treaties and the Constitution of the United States would be different in different
states.
c) Structuralist approach: The Court reached their decision by taking into account the
structure of our federal government and the effects that multi-state decisions would
have on our Constitution.

d) Uses of Judicial Review

SLAVERY
State v. Post (1845) pg. 423
F: NJ. Appeal of constitutional attack on slavery.

CON LAW I Outline Page 10


ROL: Slavery is not inconsistent with a constitutional declaration that all men are by
nature free.
R: Because no one is absolutely free. In a civilized state, all residents give up a portion of
their freedoms in exchange for the benefits of civilization. Freedom must be looked at in
the context of its society. At the time the NJ constitution was adopted, slavery was
accepted. The court is not inclined to change that view.
Class notes:
The argument that slavery is outlawed by the const. is:
Based on text all men are equal
Intent We the people who is included under we
There us also an argument that the C did not abolish slavery because in Article I it refers
to free men and 3/5 regarding property right. Also Article IV Section 3 refers to
delivering up fugitive slaves. The NJ const. did not have these slave provisions. Also an
argument is that if the framers of the C intended to abolish slavery, they would have done
so expressly in the cont. In this case the court took the view that this issue should be left
for the political process. In that sense it was a case in the favor of judicial restraint.

Seeing that there is slavery, the broad language of the preamble must not conflict,
and must not be taken literally to include all people
Court must assess the political context in interpreting
o 1804 plan for gradual abolition was meant just that, gradual, not to end
now

1787 N.W. Ordinance that states north west of the Ohio River were to be free states.
Missouri Compromise (1820) that they let Missouri have slavery
1850 Compromise
1854 KS. Neb. Compromise

Dred Scott v. Sandford (1857) pg. 427


F: Slave sued for his freedom after having been taken to a free state and returned to a
slave state.
ROL: Individuals of the Negro race are NOT considered to be citizens in the
Constitutional sense.
R: Because Citizens for Constitutional purposes, refers to sovereign people of the US
as they were understood to be at the time of the constitutions adoption. At that time,
blacks were considered to be an inferior class, one having no inherent liberty or property
rights. A state cannot confer rights as a US citizen by virtue of state law. The slave was
not considered a citizen of a state in the US in the constitutional sense and was not
entitled to sue in US courts.
Class notes:
This opinion states a federal rule that blacks are not US citizens and do not have the
rights that come with that. Since there was no jurisdiction, there is an argument that the
court had no reason to go on in the opinion. The opinion goes on to say that Congress
does not have the power to outlaw slavery in the territories. [See Article 4 section 3
clause 2 this seems to give Congress the power to abolish slavery]. AT the same time
this may be limited by something like the 1st Amendment also an argument may be that

CON LAW I Outline Page 11


that clause only applies to territories at the time of 1787. The court says that abolishing
slavery would have been a violation of Due Process by taking peoples property away.
This case is not judicial restraint in that it reached out to say that Congress did not have
the power to abolish slavery. This was the first time since Marbury that the SC struck
down a statute.

Calder v. Bull Pg. 69


This case deals with issues of natural (moral) law and how that should fit in with statutes.
What would happen where moral law conflicts with constitutional law example of
slavery if you have a judge who didnt believe in slavery or today a judge who
doesnt believe in capital punishment.

THE POWER OF REPRISAL: POLITICAL CONTROL OF THE SUPREME


COURT
1. Constitutional Amendment Under Article V you need 2/3 of both houses and
of the states. This purposefully made it difficult to pass an amendment to the C.
2. The power to appoint The President appoints SC justices at the advice and
consent of the Senate. Appointments are not always a guarantee of political
control.
3. Impeachment No SC Justice has ever been removed from office in US history.
4. Life Tenure This is supposed to promote judicial independence.
5. Controlling Sitting Judges: informal mechanisms and self-imposed limits.

e) Limitations on the Powers of the Federal Courts

Why discourage amendments?


Stability
Want widespread agreement, more people can participate in political process

27th amendment is it a constitutional amendment? Only congress can determine, SC can


not adjudicate political questions

What is compensation for the services?

How about the elapsed time?

*Constitution was in violation of the Articles of the Confederation

- Exception clause literally Exception is what it means, cant take away all appellate
jurisdiction; article III; but then, could be read in context which would assumed that court
has jurisdiction over most federal question cases

Ex parte McCardle (1869) pg. 78


(Art. III Sec. 2 states that in all cases not falling within the Supreme Courts original
jurisdiction (but falling within the federal judicial power) the Supreme Court shall have

CON LAW I Outline Page 12


appellate jurisdiction both as to law and fact, with such exceptions, and under such
regulations as the Congress shall make.)

F: D wrote articles in a newspaper and was charged for writing libelous articles. D
appealed from a denial of habeas corpus to the SC, but Congress passed an act forbidding
the Court jurisdiction. In 1867 an act was passed which gave the courts jurisdiction to
hear cases involving habeas corpus where persons were held in violation of their
constitutional rights and gave appellate jurisdiction to the SC. After arguments were
heard in this case Congress repealed the 1867 Act.
ROL: Although the SC gets its appellate jurisdiction from the Constitution, the
Constitution also gives Congress the express power to make exceptions to that appellate
jurisdiction. Congress possesses the authority to eliminate appeals to the Supreme Court,
which it had created in the Act establishing the habeas corpus.
R: Because the exceptions to the SC jurisdiction are express.
Class notes:
After the Supreme Court heard arguments in the McCardle case, but before it handed
down its decision, Congress passed a law repealing the portion of the 1876 Act which
allowed appeals to the Supreme Court. Thus Congress purported to deprive the Supreme
Court of its right to decide the McCardle case and any other habeas corpus case coming
to it by appeal from the circuit court. The Supreme Court upheld Congress restriction on
the Courts jurisdiction. The opinion noted that the appellate jurisdiction of the Supreme
Court is conferred with such exception and under such regulations as Congress shall
make. The limitation enacted by Congress here was such an exception. Therefore, the
Court concluded, it had no jurisdiction to decide the case.
Limited Significance: Thus McCardle does not by any means stand for the proposition
that Congress may strip the federal courts of in their entirety of the right to issue habeas
corpus relief; such congressional action would probably be a violation of the prisoners'
Fourteenth Amendment right to due process.
Based on article III section 1:
The following is one type of argument:
Lesser included: the greater power includes the lesser.
The greater power of Congress to eliminate the lower federal courts entirely includes the
power of Congress to do what it wants to federal court jurisdiction.
This may be attacked by disproving the premise (greater power).

Pp 85-112

f) Advisory Opinions, Standing, and Political Question Doctrine


Standing
Case or Controversy
The SC cannot decide on an issue because they think it is interesting, there must be some
personal stake involved (standing). This achieves three things:
1. Judicial Restraint
2. Ensures that constitutional issues will be resolved based upon concrete disputes
and not hypothetical situations.
3. Promotes the ends of individual autonomy

CON LAW I Outline Page 13


Advisory Opinions - These are actions that do not grow out of a case or controversy.
These opinions could be to give advice. The USSC may not issue such opinions. Some
state SCs may.

If there is no standing, then there is no judicial review.


Allen v. Wright (1984) pg. 87
F: Class action brought by parents of black school children against the IRS for failure to
deny tax-exempt status to schools that discriminate based on race. Ps sought declaratory
and injunctive relief the court of appeals held in their favor.
ROL: One does not have standing to sue in federal court unless he can allege the
violation of a right personal to him.
R: Because the standing doctrine prohibits a litigants raising another persons legal
rights. A P must allege personal injury fairly traceable to the Ds allegedly unlawful
conduct and likely to be redressed by the requested relief. The injury must be distinct and
not abstract. Here the Ps allege that (a) they are harmed directly by the fact that the
federal government is funding racial discrimination and (b) that the tax exemptions to
racially discriminatory private schools impair their ability to have their public schools
desegregated. The abstract injury (a) is not recognized. Standing only exists for those
persons who are personally denied equal treatment by the discriminatory conduct.
Otherwise people across the nation could be part of this action. The second allegation (b)
is without standing because the injury is not fairly traceable to the Government conduct
that the Ps challenge as unlawful. The line of causation is attenuated at best. It is entirely
speculative as to whether denying these exemptions would change the discriminatory
practices thus allowing for desegregation. The links in the chain of causation are too
weak. This requirement of causation goes to separation of powers. Otherwise the SC
would basically be giving an advisory opinion.
Dissent (Brennan): The causal link does exist elimination of tax-exempt status for
racially discriminatory private schools serves to lessen the impact of efforts to defeat
segregation.
Dissent (Stevens and Blackmun): Considerations of tax policy, economics, and logic all
confirm the fact that the Ps injury is traceable to the government. The fact that there is
separation of powers should not deny the Ps their remedy. A complaint should be
dismissed when the P presents a nonjusticiable issue, or seeks relief that a court may not
award, but not because the P lacks a stake in obtaining that relief and hence has no
standing.

Standing requirements:
A) Injury
B) Traceable/Causation
C) Redressability need to redress injury

Reasons why we have standing:


A) Volume of lawsuits
B) Autonomy
C) Collusion/vigorous advocacy

CON LAW I Outline Page 14


What was the personal injury that the Ps said they suffered?
Affirmative action case the P only has to show that the process was not fair, and does
not have to show that he would have been accepted to the medical school but for the
policy of affirmative action. (Backey and Jacksonville case pg. 106?) These are 2
different ways of looking at what you have to prove and how you characterize the injury:
1. Actual injury
2. The process was not fair
In most cases you dont get the benefit and then you claim an injury.
Lujan v. Defenders of Wildlife (1992) pg. 92
F: ESA Endangered Species Act. This Act was originally construed to apply
everywhere and then later only to the US and the high seas. Wildlife organizations filed
this action against the Sec. of the Interior to call for the applicability of the ESA to all
habitats and expand the geographic scope. Court of Appeals found the Ps had standing.
ROL: Only individuals who have suffered concrete harm have standing to seek judicial
review of agency rules.
R: There are three elements to Standing:
1. P suffered injury in fact concrete and actual
2. Causal connection between injury and conduct complained of must be traceable.
3. It must be likely as opposed to speculative that the injury will be redressed by
a favorable decision.
The burden is on the party invoking federal jurisdiction to establish these elements. When
the P is not himself the object of the government action or inaction, standing is not
precluded, but it is substantially more difficult to establish.
III: Ps claim they will travel to certain areas and will be directly affected for inability to
observe certain species. The Court says this injury is not imminent since they have no
set plans to travel.
Ecosystem nexus argument - that we are all part of the same ecosystem is invalid because
the P must be in the affected area specifically and not just in the vicinity.
Animal nexus argument Anyone with an interest in studying animals can sue. This
argument is beyond all reason.
Question of redressability even if the Sec. of Interior put out a new regulation requiring
consultation of foreign projects as the Ps demand, it would not necessarily provide a
remedy because it is unclear whether the funding agencies are bound by the Secretarys
regulation. Also, the agencies only supply a fraction of the funding.
IV: The Court of Appeals held that there was standing because of procedural injury
which it held satisfied the injury in fact requirement by congressional conferral upon
all persons of an abstract right to have the Executive observe the procedures required
by law. We reject this view. This does not satisfy an Article III Case or Controversy.
The province of the Court is to decide on the rights of individuals Marbury v.
Madison. Even when a statute denominates an individual right, the concrete injury
requirement must remain.
Concurring (Kennedy): Since there was no concrete injury I would not have addressed
redressability.
Conscurring (Stevens): I agree in judgment because the section of the act does not apply
in foreign countries, however the Ps do have standing because there was harm in that
their interest in this study was harmed. Regarding redressability, it is unlikely that the

CON LAW I Outline Page 15


funding agencies will not be bound by the regulation if changed [what about the fact that
the finding from these agencies is only fractional].
Dissent (OConnor and Blackmun): Requiring that the Ps have set plans to travel is
just a formality. Also to say that the distance mitigates the damage done on someone
studying a species from across the world is irrelevant. Genuine issues of fact have been
raised on the issue of redressability.
Class notes:
Canker hypo in FL why shouldnt I have standing to challenge the wholesale slaughter
of lots of trees in a part of Florida when they are coming into my backyard to cut one
tree. How much power does Congress have.
III(b) is not the majority that is the part that deals with redressability.
For Thursday we will got through IIA.

Standing how comfortable are we with judicial review? There must be in jury,
causation, and redressability. How do we define when one has suffered an injury?
How you characterize the injury is important.
Who has power over standing requirements? Exclusively the courts or Congress? See the
Lujan case. How much control do we want Congress to have over setting the standard
requirements? This is supposed to be a doctrine of judicial restraint. Here the court is
saying that they control this doctrine of judicial restraint. This doctrine comes up a lot in
litigation.

Prudential Standing Newdow


Political Question Doctrine


CB 112-118, 121-133
It is not common for a court to be dismissed because of this doctrine.
Political issues with constitutional dimensions generally are fair game for judicial review.
Political (non-justiciable) questions are issues, which may have constitutional dimensions
but are of a nature that they should be resolved in the political arena (i.e. foreign
relations).

Powell case pg. 124


This is either a political question, or simply an issue of constitutional interpretation. The
court is re-interpreting the constitution. Question of how to interpret qualifications.

Baker v. Carr (1962) pg. 112


(Principally known for the black letter definition of a Political Question.)
F: TN voters brought an action challenging a state statute from 1901 apportioning
members of the state General Assembly among the states 95 counties. Ps claimed their
votes were diluted as a result of population redistribution. The Ps sough a re-
apportionment or elections at large.
ROL: Reapportionment issues present justiciable questions.
R: Because Non justiciable, or political questions are those properly left to other
branches of government, either because courts do not have the resources to handle them

CON LAW I Outline Page 16


or because they are best decided through the political process. Examples include foreign
relations and the validity of enactments. The issue here, however, is whether the
guarantees of equal protection found in the constitution are being met. Thus, in this issue,
the court is being called upon to judge the constitutionality of the system under attack,
something squarely within the courts power and duty.
Dissent: The Court should not interject itself into political disagreements as it has done
here. The Court is not adequately equipped to handle reapportionment issues. Finally, the
structure of state institutions should be given great respect.
* Guarantee Clause Art. IV Sec. 4 Guaranty of a Republican Form of Govt.
Class notes:
Would a referendum violate the Guaranty Clause? Should the Court be deciding what a
Republican form of government is. They could have said that a Guaranty Clause claim is
a political question and then dismiss it. Instead they say it is an equal protection violation.

Factors that identify a political question:


A. Commitment to another branch: A textually demonstrable
Constitutional commitment of the issue to a coordinate political
department. See U.S. v. Nixon
B. Lack of standards: A lack of judicially discoverable and manageable
standards for resolving the issue.
C. Unsuitable policy determination: the impossibility of deciding the
issue without an initial policy determination of a kind clearly for non-
judicial discretion.
D. Multiple pronouncements: The potential for embarrassment from
multifarious pronouncements by various departments on one question.

Nixon v. US (1993) pg. 121


F: District Court Judge who was impeached and removed for making false statements to a
grand jury sought judicial review claiming the Senate had failed to try him according to
Art. I.
ROL: Judicial review of impeachment by the Senate is a political question that cannot be
resolved by the courts.
R: Because a question is political if the text of the constitution specifically designates a
specific political department. Regarding impeachment, Art. 1, Sec. 3, Clause 6 says that
the Senate has sole authority to try. Sole implies only the senate. The P argues that the
word try implies a judicial proceeding. This is not so, the framers did not impose for
the judiciary to get involved here.
1. Allowing judicial review would be inconsistent with checks and balances. Judicial
impeachment was designed to be the only check by legislature on the judicial
branch. Judicial impeachment was meant to regulate the judiciary to let them
review it as the final authority defeats the purpose.
2. Lack of finality
3. How do you provide a remedy for an overturned impeachment?
Class notes:
Nixon argues that it must be the Senate who must try the impeachment.

CON LAW I Outline Page 17


The idea that the court should tell the Senate how to go about the impeachment process is
unusual. Structural finality; Rehnquist Nixons argument would place final
reviewing authority with respect to impeachments in the hands of the same body that the
impeachment process is meant to regulate. Separation of powers. Textual argument:
sole specific limits. Another argument is that sole does not necessarily exclude the
Court, but rather it excludes the other House; Section 2, Clause 5 House shall have the
sole power of impeachment read together, suggests that just excludes other house, not
all other branches of government

Bush v. Gore supp. Pg. 2


Equal Protection argument. The FLSC says we need to do a recount saying the statute
says there should be a recount and the standard is to determine the intent of the voter. The
opinion did not say how to go about determining the intent of the voter. The USSC says
that there is a violation of Equal Protection because there is no standard for how to
determine the intent of the voter. How much authority should the federal government
have through the USSC? The USSC is telling the FLSC that they got it wrong with regard
to FL law. Article 2 Sec. 1 Clause 2 saying the concurrence says that the FLSC usurped
the legislatures role. [look at the issue of standing IIB1 on Tues. and for Thursday:
IIB2 and IIC1 and IIC2A]
Two main issues (besides standing):
1. Equal protection
2. Article II Judiciary v. Legislature
Did Bush have standing to raise the equal protection claim?
According to the court there was a violation of Article II (sec. 1 clause 2) of the
Constitution. There is no right of individual citizens to elect the President. This clause
leaves it up to the state in terms of how to do the election (in such a manner as the
legislature directs). The SC holds that the FLSC violated Article II in that it abandoned
how the state legislature has set out by statute. The dissent argues that the State SC
should be the one to resolve the interpretation of the state statute or FL Constitution. The
majority says that the FLSC was incompetent in their interpretation and that they violated
the Constitution under Article II. Perhaps this should be treated as a political question.
Standing issue For standing, besides injury, causation, and redressability, there must be
the possibility that the issue will be litigated vigorously.
Political Question - The final authority is usually Congress in these issues. Here the
judiciary settled it. How much restraint do we want to impose on the judiciary? And how
much restraint is actually imposed? Page 114 in CB on PQ in Baker case. How much do
we want the Court to be making these Constitutional issues? What should the court
decide?

Decision/Rationale
Once a state enfranchises its citizens with the right to vote, that right becomes
fundamental, and the state must afford equal weight to each vote by all citizens
Similarly, equal protection requires that a state choosing to later disenfranchise its
citizens must do so on equal grounds so as to value each persons vote equally
FSC was unable to establish a standard to discern the intent of the voter that
satisfied the EPC

CON LAW I Outline Page 18


What constitutes a legal vote in one county does not apply in a neighboring
county, and equal protection has not been preserved by the recount procedure
Bc all votes appointing members of the electoral college cannot be counted and
certified by Dec 12, as required by fed statute, and the state recount procedure is
unconstitutional, the decision of the FSC is reversed and remanded
Rehnquist the states role in the presidential election maintains a uniquely
federal character

Standing
Not mentioned in case. Why?
Would have been brought up by another citizen?
Court had to intervene to solve a crisis regardless?

Political Question
Who has power to resolve dispute for president?
o Baker 12th amendment, textual commitment, left to Congress to resolve
disputes about electoral college and votes
o

pp. 133-135
Ripeness and Mootness
A case is not ripe when it is brought too soon, and it is moot when it is brought too late.
There is an exception to mootness (Roe v. Wade) where the cases are capable or
repetition. Voluntary cessation of allegedly unlawful conduct does not make the case
moot.

II. THE SEPARATION OF POWERS


Pp 331-336
Distribution of powers
The separation of powers is not rigid there is an overlap.
Purpose of check and balances:
1. Division of labor efficiency
2. Prevention of tyranny

The Mass. State Constitution (which is older than the US Con.) calls for a clean and rigid
division of powers with little overlap. Madison says that you have to mostly worry about
Congress as the most powerful branch.
Attacks on separation:
1. Separation interferes with democratic process by preventing popular majorities
from bringing about change
2. Instead of solving problem of factions it aggravates it by allowing well-organized
private groups to block necessary regulation.

The Court has not considered separation of power cases nonjusticiable (political
questions).

CON LAW I Outline Page 19


Depending upon which branch of government you believe is most dangerous, your
dilemma will differ with certain decisions.

FRAMERS INTENT
Which of 3 branches should we be most worried about? How about the framers?
o Executive

Executive Authority:
CB 336-352, Supp. 40-45
Presidential Seizure
Youngstown Sheet & Tube Co. v. Sawyer (1952) pg. 336
(The President may not make laws; he may only carry them out)
F: Steel workers had labor disputes with their companies and threatened a work stoppage.
The President believed this would jeopardize national defense and the availability of
steel. President Truman (via an order to his Secretary of Commerce) seized the nations
steel mills. The mill owners argued that this was lawmaking which was a legislative
function given to Congress by the Constitution and not to the President. District Court
ruled that the seizure cannot stand.
I: Whether it is within the Presidents powers to seize the nations steel mills absent any
Congressional enactment.
H: No, affirmed.
R: Because absent a Constitutional provision or statute (Congressional enactment) the
Presidential function is to execute and not to make law. The argument that the C gives the
Prez the power to seize as Commander in Chief of the Army and Navy does not fly
because these are privately owned businesses.
ROL: It is NOT within the Presidents powers to seize private property (the nations steel
mills) absent a Congressional enactment.
Frankfurter concurring: Our system of government was designed to put restrictions and
safeguards on the power within each branch. Congress has withheld the power from the
President by not granting him the power to seize the mills.
Jackson Concurring: Only when Congress has enacted a provision supporting the
President would such a seizure be valid. The opinion goes on to dismantle the
governments argument based upon the 3 clauses that it cites from the constitution. There
are three categories:
1. When the Prez acts with the authorization of Congress maximum power
2. When the Prez acts and Congress is silent - ok power
3. When the Prez acts and Congress is expressly in disapproval lowest power
The President has some independent powers even without Congressional approval.
When the Congress specifically disapproves of the Presidents action, he can rely only on
his powers as enumerated in the Constitution. This action falls into the third category
(with the rejection of a similar Act that would have given the Prez power to negotiate for
unions), and the President has overstepped his ability to move independent of Congress.
Douglass Concurrence: The 5th Amendment can only be read to give condemnation power
to Congress. Only Congress can compensate so only they can condemn.
CJ Vinson Dissenting: The Presidents action was legitimized by the fact that it was
invoked to avert a national disaster. We must look at the context of this action. The

CON LAW I Outline Page 20


President has the duty to take care that the laws be faithfully executed. During WWI
the President seized the coalmines. Presidents have taken prompt actions to enforce the
laws and protect the country whether or not Congress happened to provide in advance for
the particular method of execution.
Class notes:
The Korean War was going on. Truman could have kept the steel mills open on the
owners terms by authorizing wage increases, however he did not do this. The issue is
that it is not that the Federal government cannot seize the steel mills. The 5th Amendment
allows Congress to take property and then to compensate. The issue is that while
Congress can do this, the President does not have this power based on the Constitution.
The majority says that there is no statute authorizing the Prez to seize. The Prez argues
this is an executive act; Black rejects this saying that this is rather a legislative act (for
Congress). How can we tell the difference in terms of what kind of act this is? This was
not the enforcement of a Congressional Act. Perhaps this is a policy. For Black, earlier
Presidential seizures were irrelevant.
For Frankfurter, the past seizures are indeed relevant. It is like the argument in
McCullough. History counts because we can recognize that some body other than the
court can interpret the Constitution if we look at the fact that Congress has upheld
Presidential seizures in the past. The problem with the argument of historical gloss is that
you still have to explain the interpretation. Under Jackson, this is a case where Congress
has invalidated the seizure and Congress is not silent because Congress did reject a
similar Act to say that they were not just silent, but they have dealt with the issue. Even if
something is in the 3rd category we must analyze whether the Constitution grants the
Prez the power such that the Act of Congress is unconstitutional.

Dames & Moore v. Reagan (1981) pg. 348


F: President Carter, in exchange for release of hostages, issued an order barring court
action against Iran or its citizens. Ps assert breach of K against Iranian banks and brought
action to declare Presidential order unconstitutional. Congress was silent on the order.
Lower courts upheld the order. Reagan ratified the order.
ROL: A Presidents may order to settle legal claims when the order is ancillary to major
foreign policy issues and Congress Acquiesces. Congress tacitly approved the action by
approving similar actions.
R: Because Congress has passed the IEEP Act which gives the Prez similar powers for
similar situations. Congress cannot be expected to legislate for every contingency. There
was long-continued practice, known to and acquiesced in by Congress.
Class notes:
Comparing this case to Youngstown in Youngstown Congress did not authorize the Prez
to seize the steel mills. The issue is whether the President has the authority to suspend
claims by US citizens against Iran unless you view these claims as transactions which
is listed in the IEEPA. Is stopping lawsuits considered a transaction? It is unclear.
Which Jackson category did this case fall under?
There may be an argument that this is a category 3 because of the fact that Congress
passed acts such as IEEPA but did not authorize suspension of claims. You could
analogize this to Youngstown and say that Congress has thought and dealt with the issue
and specifically did not authorize the power to suspend claims. The 2nd category would

CON LAW I Outline Page 21


only apply where Congress has actually not thought about an issue. It may be the 2nd
category where Congress is silent, but this probably is not that category because it is not
as if Congress has not at all thought about this. There may be an argument that it is the 1st
category since Congress has authorized similar acts by the President and has also
historically Presidents have been able to do similar things with the implied will of
Congress.
Limited scope: The Court carefully stressed the limited scope of its holding. It was not
holding that the President has Constitutional authority to settle or suspend all claims;
the Court was simply deciding that where such settlement or suspension is a necessary
incident to the resolution of a major foreign policy dispute and Congress has acquiesced
in that type of presidential action, the action will be deemed within the Presidents
Constitutional authority. Also note that Congress acquiescence is a factor in determining
whether the President was acting within his authority. But is not a decisive element
without other factors.

Pg. 40 supplement Military Tribunals


Does the Prez have legislative authority to setup military tribunals? Joint Resolution of
Congress.

DOMESTIC AFFAIRS
Executive Privilege, Impeachment, and Immunity
US v. Nixon (1974) pg. 352
Art II sec. 1 states in pertinent part that the executive power shall vest in the President.
Art II sec. 3 provides that the President shall take care that the laws be faithfully
executed.
(The notion of executive privilege is first introduced.)
F: Employees of the reelection committee for Prez Nixon broke into the DNC at the
Watergate Hotel. There was a subpoena requiring the Prez to produce certain tape
recordings relating to conversations with aids and advisors. The Prez sought a motion to
quash the subpoena and it was denied. This is a review of the denial of that motion.
ROL: Absent a claim of need to protect military, diplomatic, or sensitive national security
secrets, an absolute presidential privilege of immunity from judicial process under all
circumstances does not exist.
R: Because:
1. It is up to the judiciary to say what the law is.
2. The needs of the judicial process (evidence) outweigh presidential privilege
Class notes:
Executive Privilege, Impeachment, and Immunity
US v. Nixon (1974) pg. 352
Art II sec. 1 states in pertinent part that the executive power shall vest in the President.
Art II sec. 3 provides that the President shall take care that the laws be faithfully
executed. the president argues that he needs to get candid advice from his
subordinates and so if his confidential conversations were exposed, hed be
INEFFECTIVE. You cannot have checks and balances if one branch is hobbled. To be
effective you need candor from your subordinates (* this argument most persuades The
Court).

CON LAW I Outline Page 22


The Constitution didnt really say anything, though, granting President executive
privilege.
so you could argue that this is a big deal for the president because its really the first
case in history that agrees that there is an executive privilege at allbecause its never
set forth in the Constitution explicitly.
What even is executive privilege? Is it the privilege to not be subpoenaed? Is it the
privilege to shield Prez. Communications with his subordinates?
(The notion of executive privilege is first introduced.)
F: Employees of the reelection committee for Prez Nixon broke into the DNC at the
Watergate Hotel. There was a subpoena requiring the Prez to produce certain tape
recordings relating to conversations with aids and advisors. The Prez sought a motion to
quash the subpoena and it was denied. This is a review of the denial of that motion.
ROL: Absent a claim of need to protect military, diplomatic, or sensitive national security
secrets, an absolute presidential privilege of immunity from judicial process under all
circumstances does not exist.
R: Because:
3. It is up to the judiciary to say what the law is.
4. The needs of the judicial process (evidence) outweigh presidential privilege
Class notes:
Argument 1 (too strong) Im immune
Argument 2 (distinction from above, less strong, courts like)- I invoke exec.
Privilege.
(prez. has no constraints in doing so by either the SC or the people- except hed be
scrutinized).
Should the President have executive privilege? Is there anything in the Constitution? The
constitution does not refer to presidential privilege. The Prez may argue that the fact that
the C is silent on it does not mean that he does not have that power it can be implied in
the C (like the power to setup a bank). This opinion does recognize that the Prez has
Executive Privilege, however he did have to turn over the tapes because the case was
against those employees who broke into the Watergate. This is like the other Nixon case
where the Court says it is not for the Court to decide on the Constitutional interpretation.
Here the Court says it is not up to the Executive branch to decide the scope of what is
protected by the Constitution.

However, Nixon could have just fired the Special Prosecutor and eventually he did,
which was Black Saturday.

1. Absolute privilege: Doctrine of Separation of Powers precludes JR of a


presidents claim to [absolute privilege]. Absolute privilege says there
absolute confidentiality to ALL presidential conversations. = Im
immune
2. If not in absolute privilege, at least the court must consider that the
privilege prevails over the subpoena duces tecum (for policy reasons of
danger of exposing confidentialities of Pres.) I invoke exec. Privilege.

Courts answer: (citing Marbury)

CON LAW I Outline Page 23


1. President does not say the law. Marbury establishes power of judicial
review(which also lets SC establish why someone has standing) and
Marbury emphatically reaffirmed that, it is emphatically the province
of the judicial department to say what the law is.

Plus the judiciary cannot share its law-dictating powers with the
Executive. This cuts too deeply into due process, the court replies.
Because it undercuts the SCs defense too much and also to the public,
if it undercuts the prosecutor. So theres a specific need for the tapes.
And also theres nothing special really about these tapesits just a
general need for confidentiality and candor vs the SPECIFIC need of SC
to use them.
2. Plus the submissions will be in-camera and so will be protected by the
District Court and the protective mandates imposed judge-wide
thereon. Therefore, it is hard to swallow the argument that the
Constitutional reason for privilege overrides the subpoena.

Clinton v. Jones (President is liable for civil damages that arise out of non official
activities that have taken place before he has taken office.) WEIGHING AND
BALANCING OF HARMS. SC MAKES CALCULATIONS ABOUT WHAT WILL
IMPAIR THE PRESIDENTS EFFECTIVENESSHOW DO THEY KNOW? THE
FACT THAT THEY EVEN ACKNOWLEDGED IMMUNITY IN THIS CASE MEANS
THEY REALLY ACKNOWLEDGED THAT IT EXISTED.
Facts: Jones alleged that while President Bill Clinton was Governor of Arkansas, he had
sexually assaulted her. She also alleged that because she did not actively participate in
these sexual advances, she was demoted from her state job.
Issue: Should a President be subjected to a civil suit for purely unofficial acts that have
occurred before he became president?
Yes. The President can be held liable in civil damages for activity that was non-related to
his specific job function.

Should the President have executive privilege? Is there anything in the Constitution? The
constitution does not refer to presidential privilege. The Prez may argue that the fact that
the C is silent on it does not mean that he does not have that power it can be implied in
the C (like the power to setup a bank). This opinion does recognize that the Prez has
Executive Privilege, however he did have to turn over the tapes because the case was
against those employees who broke into the Watergate. This is like the other Nixon case
where the Court says it is not for the Court to decide on the Constitutional interpretation.
Here the Court says it is not up to the Executive branch to decide the scope of what is
protected by the Constitution.
Clinton v. Jones (President is liable for civil damages that arise out of non official
activities that have taken place before he has taken office.)

CON LAW I Outline Page 24


Facts: Jones alleged that while President Bill Clinton was Governor of Arkansas, he had
sexually assaulted her. She also alleged that because she did not actively participate in
these sexual advances, she was demoted from her state job.
Issue: Should a President be subjected to a civil suit for purely unofficial acts that have
occurred before he became president?
Yes. The President can be held liable in civil damages for activity that was non-related to
his specific job function.

Legislative Authority
Non-Delegation
Practical necessities
Essential powers
Intelligible principles
An Act delegating authority must be specific and limited and provide guidance. Hypo: a
statute delegating to the FCC the authority to enact rules governing e-commerce. This is
too broad. Congress must provide a standard. They can add in the public interest.
The Supreme Court uses two approaches in evaluating legislative authority. 1)
Formalist- this approach emphasizes clear rules that demarcate separate spheres of
authority. 2) Functionalist- fluid approach that prohibits aggrandizement of power

Myers v. US pg. 379


(President power of removal is confined to purely executive officers)
Upheld that Prez can remove Postmaster with the advice and consent of the Senate.
The Prez can rely on the Take Care clause to argue that it is upon him to execute the laws
and not his subordinates.

Humphreys: The presidential right of the Prez to remove members of the FTC. This was
held unconstitutional. There are 2 distinctions:
1. The Postmasters function is executive
2. Here the FTC removal had no limitation, such as consent of Senate), as the
postmaster removal had.
Some view independent agencies as the 4th branch.
The Court distinguished Myers and said that the office of a postmaster is so essentially
different from the office now involved that the decision in Meyers does not apply. In
Myers the office was purely executive in nature. Here the Commission acts in part quasi-
legislative and in part quasi-judicial. Thus the holding in Meyers only applies to purely
executive officers.

INS v. Chadhu (1983) pg. 370


F: Congress vetoes the AGs recommendation to suspend the deportation of a non-
American.
ROL: A one-house veto by Congress, which is not subject to bicameral and Presidential
review, is NOT constitutional.
R: Because the Constitution specifically refers to certain circumstances where there can
be one-house action without being subject to Presidential veto, and this is not one of

CON LAW I Outline Page 25


those exceptions. The veto provision is unconstitutional because it extends the scope of
authority beyond that set forth by the C.
Powell Concurring: This should have been decided more narrowly on the basis of
separation of powers and the fact that Congress acted beyond the scope of its
Constitutional powers by doing an act that has traditionally been left to other branches,
and should not have dealt with the constitutionality of Congressional vetoes. Congress
act was not legislative.
White Dissenting: The majority should have decided this on separation of powers and not
dealt with Congressional veto. Congressional veto is a very important tool:
1. Allows Prez and Congress to resolve major differences
2. Assures accountability of independent regulatory agency
3. Preserves Congress control over lawmaking
The C does not directly prohibit or authorize legislative veto this should not be inferred
as disapproval. The flexibility to use this tool is necessary. Only Bills and their equivalent
are subject to the requirements of bicameral passage and presentment to the Prez. The
legislative veto is an exercise of quasi-legislative power. The AGs suspension of
deportation is equivalent to a proposal for legislation.
Class notes:
If an act does not impose a duty or grant rights, then it is not legislative and does not
require bicameralism and presentment. If rights and duties are altered, then you do need
bicameralism and presentment. The court here says the act by Congress was legislative
and that Chadhas rights were altered. However we can argue that the statute to suspend
the deportation did not actually alter any rights or duties, in which case you can argue
against the majority that this was not really legislative and does not require bicameralism.

Administrative Agencies; where do they fit in the separation of power scheme?


1. Art. II vest the executive power in the President alone, not in a subordinate
officials.
2. A unitary executive based on the efficiency justification for the separation of
power.
3. Need to ensure expeditious, centralized, and coordinated authority in law
enforcement

Browsher v. Synar (1986) pg. 381


(Retention by Congress of the right to remove an executive officer for certain specified
types of causes converts that officer into an agent of Congress.)
F: The Comptroller, while appointed by the Prez, is removable only by Congress. The
Comptroller reviewed all public funds received and disbursed by Congress. Under the
Gramm-Act, he had the authority to independently estimate budget deficits and targets,
and make budget cuts. A three-judge panel held that the act was unconstitutional.
ROL: It is NOT Constitutional (in terms of separation of power) for the Comptroller, who
performs executive functions, to be removable only by Congress.
R: Because to permit the execution of laws to be vested in an officer answerable only to
Congress would reserve Congress control over the execution of laws. The Constitution
does not allow for Congress to execute laws. Congress cannot grant to an officer under its
control what it does not posses. This would be like allowing a Congressional veto. The

CON LAW I Outline Page 26


Comptroller has always been viewed as an officer of the legislative branch as such he
cannot retain executive powers.
Now we must consider whether this Act grants him executive powers. Interpreting a law
enacted by Congress to implement the legislative mandate is the very essence of
execution of the law. The Comptroller must exercise judgment and interpret while
applying the Act these are executive functions. By Congress controlling the ability to
remove the Comptroller, it has intruded into the executive function. Act is
unconstitutional.
Stevens and Marshall Concur: Agree that the Act is unconstitutional, but not because of
the removal power by Congress, or the fact that the Comptroller is given executive
functions. Rather, since the Comptroller sets policy that binds the nation, he should be
subject to the Presentment Clause (Presidential veto) under Article I of the Constitution.
Indeed the power granted here is legislative.
White Dissenting: Accept the proposition that the Comptroller performs some executive
functions. DO not accept that Congress having sole removal power is analogous to the
impermissible execution of law by Congress itself. This is very different than the
Congressional veto being struck down in Chadha. The Comptroller may only be removed
through a joint resolution passed by both Houses and signed by the Prez. This satisfies
bicameralism and presentment. Congress having some control over the Comptroller does
not make the Act unconstitutional. The fact that Congress had removal power was not to
make the Comptroller dependent on Congress, but rather to render him independent in the
federal establishment.
Class notes:
The Act created by Congress fails for three reasons: first the Act uses the Comptrollers
executive powers; second, executive powers may not be vested by Congress in itself or it
agents, because congress is limited to legislative rather than executive functions; third,
because Congress can remove the Comptroller, he is an agent of Congress.

Washington Airports Authority v. Citizens against aircraft noise (1991) pg. 399
F: The Metro Authority per a Congressional Transfer Act now ran The DC airports
previously run by the Feds. There was to be a Board of Review made up of members of
Congress with veto power. The statute was contested on the grounds that it violated
separation of powers.
ROL: It is unconstitutional as a violation of separation of powers to have a review board
with veto power made up by members of Congress to oversee a Transfer Act enacted by
Congress.
R: Because Congress may not exercise executive powers. Even when they exercise their
constitutional legislative powers they must do so according to the procedures specified in
Article I. The Act is unconstitutional whether the authority is executive or legislative.
Dissent: There is no issue of separation of powers since this is state law. The Board
would not exist but for Congress. Congress has no power to remove anyone from the
board there is no violation here.
Class notes:

Notes on Pg. 442

CON LAW I Outline Page 27


Meyers v. U.S pg. 379
(President power of removal is confined to purely executive officers)
Facts: A congressional statute provided that the postmaster shall be appointed and may
be removed by the president and with the advice and consent of the Senate. President
Wilson attempted to remove Myers, a postmaster, before his term expired and the Senate
opposed.
Issue: Whether the removal of a purely executive official will be permitted without the
advice and consent of the Senate.
Yes. The removal was lawful, because the restraint imposed by congress was an
unconstitutional attempt to limit the Presidents removal power under Art. II. The Court
based it on 1) the act of removal is executive in nature and must only be performed by the
President; 2) under the take care clause Art II sec 3, this power is solely in the President
not his subordinates; 3) the executive power is vested solely in the President.

Humphreys Executor v. U.S. pg. 379


involved a statute providing that members of the Federal Trade Commission could be
removed by the President for inefficiency, neglect of duty, or malfeasance in office.
Legislative history shows the statute was designed to entrust regulatory decisions to a
body of non-partisan experts, insulated from political pressure. Roosevelt removed
Humphrey from office, contending not that removal was justified by one of the statutory
conditions for removal, but that the limitation on the removal power was unconstitutional
under Meyers. The President claimed that the Meyers Court held that subordinate
officials served at the pleasure of the President.
Issue: Whether the Meyers decision could be read to allow the firing of subordinate
officials at the Presidents will.
No. The Court distinguished Myers and said that the office of a postmaster is so
essentially different from the office now involved that the decision in Meyers does not
apply. In Myers the office was purely executive in nature. Here the Commission acts in
part quasi-legislative and in part quasi-judicial. Thus the holding in Meyers only
applies to purely executive officers.
Class notes:
Accountability -

Morrison v. Olson
(As long as the executive retains some control of independent inquiries the separation of
powers principle is not violated.)
Rule: The Court upheld the Independent Counsel Act because it did not violate the
separation of powers by increasing the power of one branch at the expense of another.
Instead, even though the President could not directly fire the independent counsel, the
person holding that office was still an Executive branch officer, not under the control of
either U.S. Congress or the courts.
Is the AG appointment consistent with the Constitution?
Is the removal of AG consistentis it consistent with the prior case law?
Whats the significance of the inferior officer firing ability of Congress?
o Inferior= limited mandate, position ends _____
o Is the president going to choose a vigorous investigator?

CON LAW I Outline Page 28


o The Senate may pick someone more vigorous, but it may be more heavily
politicized
o So this is the advantage of The Court, depoliticizing.
Facts: The statute in Morrison required the Attorney General to investigate any
allegations of wrongdoing against certain high level members of the Executive Branch
(including members of the Cabinet) and to apply to a special federal court (The Special
Division) for the appointment of a special prosecutor if he found reasonable grounds to
believe that further investigation or prosecution is warranted. Once the special
prosecutor was appointed, she could only be removed by the Attorney General, and only
for good cause.
Issue: Whether the statute in Morrison violated the separation of powers principle
because it restricted the President from firing at will.
No. The Morrison Court determined that the special prosecutor was an inferior officer
rather than a principal officer following the same reasoning in Meyers and
Humphreys Executor. The Court held that neither the removal provisions nor the act
taken as a whole so restricted the Presidents power as to violate the separation of powers
principle. Because the Attorney General could terminate the special prosecutor for good
cause the Executive Branch retains ample authority to assure that the counsel is
competently performing her statutory responsibilities.
Dissent: Scalia contended that the separation of powers principle required that the
President maintain complete control over the investigation and prosecution of violations
of law. Since even by the majorities reasoning the Presidents control over the special
prosecutor was curtailed, in Scalias view the Act was clearly a violation of the
constitutionally required separation of powers.
Foreign Affairs

Buckley v. Valeo
A Federal Election Commission was created by congress and its members were appointed
as follows. Two by the President pro-tempore of the Senate, two by the speaker of the
house, two by the president. The Court unanimously struck down the method of
appointment as unconstitutional. Art II provides in pertinent part: the president shall
nominate, and by and with the consent of the Senate, shall appoint ambassadors, other
public ministers and consuls, judges of the Supreme Court, and all other officers of the
United States. . . . but congress may by law vest the appointment of such inferior
officers, as they think proper, in the president alone, in the Courts of law, or in the
heads of departments. Thus, any appointee exercising significant authority pursuant to
the laws of the U.S. is an officer of the U.S. and must therefore be appointed in
accordance with the appointments clause. The Court held that inferior officers are
different than officers of the U.S. only so far as the appointment clause applies. The
First is subject to the latter part of the clause, while the second must be nominated by the
President and appointed by the Senate.

Bowsher v. Synar
(Retention by Congress of the right to remove an executive officer for certain specified
types of causes converts that officer into an agent of Congress.)

CON LAW I Outline Page 29


Facts: Under the Balanced Budget and Emergency Deficit Control Act of 1985, the
Gramm-Rudman-Hollings Act, Bowsher , the defendant, comptroller General of the
United States, was assigned the duty of effectuating across the board spending reductions
in the federal government. Synar, the plaintiff, sued Bowsher on behalf of the National
Treasury Employees Union, which claimed that its members were injured because
Bowshers spending reductions suspended certain cost of living benefit increases. Synar
contended that the Act was unconstitutional because the Comptroller General was
authorized under the Act to exercise executive functions, violating the doctrine of
separation of powers. By separate, much older, legislation, Congress reserved to itself
the right to remove the Comptroller General from office for five specified reasons:
permanent disability, inefficiency, neglect of duty, malfeasance etc.
Issue: May Congress create an executive position i.e. the Comptroller General but retain
the right to remove the officer for enumerated causes?
No. The Act created by Congress fails for three reasons: first the Act uses the
Comptrollers executive powers; second, executive powers may not be vested by
Congress in itself or it agents, because congress is limited to legislative rather than
executive functions; third, because Congress can remove the Comptroller, he is an agent
of Congress.

The War Powers Resolution Act Pg. 410

*War powers resolution as an unconstitutional infringement on the powers of the


President
I. The Constitution gives the President the authority to introduce
armed forces into hostilities without a delegation of war
II. The Resolution is too broad, at least in cases that deal
with the repel of sudden attacks on the United States
because it allows a congressional role in areas where
such a role is constitutionally proscribed.
III. There is nothing constitutionally illegitimate or even
dubious about undeclared wars they have been fought
for 200 years. This particular resolution would require
the President to give notice of hostilities within 48
hours, whereas before he would not have to do so.

*The War Power Resolution is Constitutional

I. It merely restores the constitutional balance that has been upset by


a long period of congressional inactivity. In essence, the resolution
allows Congress to ensure that there is no undeclared war. Indeed,
to the extent that it suffers from any constitutional defect, it is in
yielding power to the president because the resolution does seem to
allow the President to wage war without a declaration in far too
many circumstances.
II. The inherent powers which opponents make reference to are really
powers thought to be inherent because the President has historically

CON LAW I Outline Page 30


exercised them, in the absence of congressional approval objection. "This,
however, does not by itself prove that Congress lacks the authority to limit
that exercise of power . . . "
*Practice under the resolution

I. The resolution did not address covert wars


II. Congress said nothing about short-term military strikes that
could be completed within 60 days and this silence has
been taken by some presidents as freedom to treat the
statutory limit as a de facto congressional permission to
commit troops abroad for up to 60 days
The resolution has also failed to stop the Vietnam type of conflict from creeping
escalation.

Why does Congress have any role at all in declaring war? War is a really big deal that can
affect peoples lives, so we dont just want the President alone to unilaterally make a
decision to go to war. Therefore we require a joint resolution by Congress and the
President. It takes both branches to go to war. Can Congress alone declare war? See
Article 1, sec. 8, clause 11. There is no mention of the President here. Can the President
veto Congress decision to go to war? Does Chadha apply here?

A joint resolution is a vote by the House and Senate and President.


A Concurrent resolution is where it is the Senate and Congress.

Some Presidents have taken the view that the resolution is not constitutional.
Treaties The President ratifies treaties with the advice and consent of Senate and
Congress. Under Supremacy clause, the treaty would override the state law.

III. THE POWERS OF CONGRESS

Commerce Clause
Gibbons v. Ogden (1824) pg. 143 (Commerce Clause first introduced)
Art I Sec. 8 cl. 3 "The Congress shall have the power to regulate Commerce with foreign
Nations, and among the several States, and with the Indian Tribes."
Art VI cl. 2 "This Constitution, and the Laws of the United States which shall be made in
Pursuance thereof; and all Treaties made, or which shall be made, under the Authority of
the United States, shall be the Supreme Law of the Land; and the Judges in every State
shall be bound thereby, any Thing in the Constitution or Laws of any State to the
Contrary notwithstanding."
Facts: Ogden acquired, by grant from the New York legislature, monopoly rights to
operate steamboats between New York and New Jersey. Gibbons began operating
steamboats in violation of Ogden's monopoly. Gibbon's boats were licensed, however,
under a federal statute. Ogden obtained an injunction in New York court ordering
Gibbons to stop operating his boats in New York waters.

CON LAW I Outline Page 31


Issue: Whether Gibbon's license under a federal statute was constitutional and thus
overruled the New York legislature grant of a monopoly based on the Supremacy Clause.
Yes. In reaching this conclusion, Marshall took a broad view of Congress's power under
the Commerce Clause. Under that clause, he said Congress could legislate with respect
to all "commerce which concerned more States than one." Commerce, he concluded,
included not only buying and selling, but all "Commercial Intercourse." Marshall
concluded that the congressional power to regulate interstate commerce included the
ability to affect matters occurring within a state, so long as that activity had some
commercial connection with another state. Furthermore, the most important portion of
his opinion, Marshall stated that no area of interstate commerce is reserved for the state
control. " This power, like all others vested in Congress, is complete in itself, may be
exercised to its utmost extent, and acknowledges no limitations, other than are
prescribed in the Constitution." In what ways might Gibbons be said to resemble the
political doctrine generally? The political doctrine is a Court created limitation that
leaves Congress with broad discretion. Like the Gibbon's Court conclusion that
ultimately left Congress with broad discretion.
Class Notes:
One issue of the commercial clause is the supremacy clause. You simply have a federal
statute versus a state statute. Clearly the deferral statute is supreme. However, the
question is whether the federal Act was constitutional and whether Congress had the
power to license the ferry. You can regulate but maybe you cannot prohibit.
There is a three-prong test here for being considered interstate:
1. Internal
2. Doesnt affect other states AND
3. No need for federal government to regulate.

For Congress to exercise the power under the clause it must be:
1. Commerce (limits and definitions)
2. Interstate (among)
3. Regulated (as opposed to prohibited)

Who should judge whether a certain situation meets these criteria so that Congress could
exercise the power to regulate commerce.

The elements to be analyzed are:


Commerce
Interstate (three-prong test)
Regulation

The Court used a pretty broad definition on all of these terms. Marshall argued that
commerce should be construed very broadly.

There may be an argument that the fact that Congress has the power to regulate
commerce, coin money, and declare war, does not mean that the states cannot also have
those powers. The reason is that later on in section 10 (article I) the C says that the states

CON LAW I Outline Page 32


cannot coin money or engage in war so why would the C go out of its way to say the
states cannot do these things unless the Congress power was NOT exclusive.

Gibbons held that there could be Federal and State power to regulate commerce
concurrently as long as the state law does not conflict with the federal law, and as long
as the scope of state regulation is not too far reaching.

Gibbons If states cannot regulate interstate commerce, how can they perform
inspections (health etc.) under the inspection laws? They would argue that this does not
fall under commerce. If this were the case then Congress would not be allowed to
regulate these inspections since they are not interstate commerce.

Dormant Commerce Clause - States power to regulate other states commerce.

Two approaches in interpreting the commerce clause;


1. Realistic approach looks at the effect of the legislation.
2. Formalistic approach which looks at the regulation itself.

Pre-New Deal
U.S. v. E.C. Knight Co. pg. 161 ("Manufacture" vs. "Commerce)
Note: between 1880 and 1937, the Supreme Court took a highly formalistic view toward
interstate commerce.
The federal government sued under the Sherman Antitrust Act to force a major sugar
refiner to divest itself of other refiners that it had recently acquired. The Court held that
Congress could not, under the Commerce Clause, forbid a monopoly in "manufacture."
The refinery was a manufacturing operation, and was therefore left to state control. The
fact that the refined sugar was eventually sold "in commerce" was irrelevant, since the
manufacturing operation's relation to commerce was only "incidental and indirect."
Protecting principles of Federalism: The Court's rationale for its holding was the
protection of state's rights to regulate local activities. The majority contended that if the
federal commerce power extended to "all contracts and combinations in manufacture,
agriculture, mining and other productive industries, whose ultimate result may affect
external commerce, comparatively little of business operations and affairs would be left
for state control."
Class Notes:
Why is manufacturing different from commerce? Commerce seems to be more about
trading and buying and selling.

New Deal
Carter v. Carter Coal Co. (1936) pg. 169
In Carter a challenge was made against the Bituminous Coal Conservative Act of 1933,
which set maximum hours and minimum wages. The Act was found to be an invalid
use of the commerce power. Production, which was what was being regulated here, was
a purely local activity, even though the materials produced would nearly all ultimately be

CON LAW I Outline Page 33


sold in interstate commerce. Nor did production directly affect interstate commerce; the
issue was not the extent of the effect produced on interstate commerce, but the existence
or non-existence of a direct logical relation between the production and the interstate
commerce. Here the court said production falls outside of commerce.
Class notes:
Here Congress was trying to regulate the wages, labor and employment conditions and
the Court found that this is not considered commerce. Here the Court said this activity
is not commerce. Commerce or not, if the activity directly affects interstate commerce it
may be argued that there can be regulation.

Dry Cleaning Hypo from syllabus: Federal government compels specific disposal of
toxic materials. Is this constitutional during the time just before the New Deal? One
argument is that Congress cannot regulate this since it is not trade or buying and selling.
This assumes that the court should be deciding what is and is not considered
commerce. In Gibbons Marshall said that the legislature should decide what is and isnt
considered commerce. How much can you leave it to Congress to determine the
appropriate relationship between the federal government and the states? We may say that
this is the type of thing where it would be ok for Congress to regulate. Next we evaluate
whether this is interstate. This is just a local business in Texas.

Champion v. Ames (1903) pg. 164 (Lottery Case "Police power")


Facts: Congress passed the Federal Lottery Act, which prohibited the interstate shipment
of lottery tickets; the Court upheld the statute. The majority began with the assumption
that lotteries were clearly an "evil" which it was desirable for Congress to regulate; since
Congress regulated only the interstate shipment of these evil articles, it could not be said
to be interfering with intrastate matters reserved for state control.
Class Notes:

Schechter Poulry Corp. v. U.S. (1935) pg. 167 (Adopting the Knight rationale that there
must be a direct and logical relationship between intrastate activity being regulated and
interstate commerce)
Facts: The National Industrial Recovery Act authorized the President to adopt "codes of
fair competition" for various trades or industries; the code regulated such items as
minimum wage and prices, maximum hours, collective bargaining etc. Schechter Poultry
was convicted on charges of violating the wage and hour provisions of the Act. Although
the vast majority of poultry sold in New York came from other states, Schechter itself
bought within N.Y. City, and resold its stock exclusively to local dealers. The
government argued that Schechter's conduct could constitutionally be reached under the
Commerce Clause, on the alternative theories that: 1) Schechter's activities were within
the "stream of commerce" of the chicken trade; and 2) Schechter's activities though
themselves completely local, substantially affected commerce.
Holding: The Court rejected both of the Government's arguments, and held the NIRA
unconstitutional as applied to Schechter. The Court reasoned that Schechter's activities
were not in the "current" or "stream" of commerce, because the interstate transactions

CON LAW I Outline Page 34


ended when the shipments reached schechter's N.Y City slaughter-houses. Nor was the
"affecting commerce" rationale applicable; what was required was a direct, not indirect,
effect on commerce. Although Schechter's wage and price policies might have forced
interstate competitors to lower their own prices, this impact was much too indirect to
allow for Congressional control.
Class Notes:
You could make a stream of commerce argument here but the court did not buy it.

Effect on interstate commerce direct vs. indirect. It is pretty arbitrary where we draw
the line.
Intent what if there is a regulation of an activity intended to be interstate commerce but
in reality it is not? Pg. 163 Coronado here Congress was able to exercise its power to
regulate.

The method of analysis is to first ask if the activity is commerce, then even if not you
look at the direct/indirect affect that the activity has on interstate commerce. You also
look at the isolated activity versus the aggregated effect.

With the child labor case, Congress wasnt really interested in regulating interstate
commerce, they wanted to regulate child labor laws this is the idea of pretext.
The problem with having state legislated ant-child labor laws: Race to the bottom
competition among states. Because of the timing issue states would lose money in
competing with other states that could still employ children at cheaper rates. Alternatively
states can pass state law a law as a compact. Congress could prohibit lottery tickets.

Shreveport Rate Case (Substantial economic effects)


In Houston E. & W. Texas Railway Co. v. U.S pg. 162
The Interstate Commerce Commission, after setting rates for transport of goods between
Shreveport, Louisiana and various points in Texas, sought to prevent railroads from
setting rates for hauls totally within Texas which were less per mile than the Texas to
Shreveport rates. The railroads countered that it was beyond Congress' power to control
intrastate rates of an interstate carrier.
Holding: Apart from the "substantial economic effects" rationale for sustaining
congressional action, Justice Holmes developed the "current of commerce" rationale.
Under this theory, an activity could be regulated under the commerce power not because
it had an effect on commerce, but rather, because the activity itself could be viewed as
being "in" commerce or as being part of the "current" of commerce.
Class Notes:

NLRB v. Jones & Laughlin Steel Corp. (1937) pg. 175


(The modern trend in the analysis of the Commerce Clause namely the Court
substantially loosened the nexus required between intrastate activity being regulated and
interstate commerce)
Facts: The Jones & Laughlin case tested the constitutionality of the National Labor
Relations Act of 1935. The case involved the NLRB attempt to prevent Jones and

CON LAW I Outline Page 35


Laughlin, a large integrated steel producer, from engaging in "unfair labor practices" by
discriminatory firing of employees for union activity. The act allowed employees to
organize and collectively bargain.
Holding: The Court held that the NLRA, as applied here, lay within the commerce power.
The Court noted that while Jones & Laughlin manufactured steel only in Pennsylvania, it
owned mines in two other states, operated steamships on the Great Lakes, held
warehouses in four states, and sent 75% of its product out of Pennsylvania. The Court
concluded, a labor stoppage of the Pennsylvania intrastate manufacturing operations
would have a substantial effect on interstate commerce. Here, the Court rejected the
manufacture v. commerce distinction promulgated in the knight case. Thus the Court
abandoned the "current of commerce" rationale and made it irrelevant whether an activity
being regulated occurs before, during or after the interstate movement. The regulated
activity need only have a "substantial economic effect" upon interstate movement.
Class notes:
The court upheld the Act as constitutional since there was such a national commerce
ingredient here. This case does not overrule the Shechter case because there wasnt the
same interstate commerce aspect. In Shechter the impact on national commerce was
remote, and here it is immediate.

US v. Darby (1941) pg. 179


(Overruling the child labor case and disregarding Congress' intent)
In Darby, the Court upheld the Fair Labor Standard Act of 1938, which set minimum
wages and maximum hours for employees engaged in the production of goods for
interstate commerce. The Act not only prohibited the shipment in interstate commerce of
goods made by employees employed for more than the maximum hours or not paid the
prevailing rates, but it also made it a federal crime to employ workmen in the production
of goods "for interstate commerce" at other than the prescribed rates and hours.
Holding: The Court first upheld the direct ban on interstate shipments; it disposed of the
argument that manufacturing conditions are left for exclusive state control, by stating that
" the power of Congress over interstate commerce can neither be enlarged nor diminished
by the exercise or non-exercise of state power." The 10th Amendment states only a
"truism" and should therefore not inhibit congress acting within their prescribed
authority. The Court also disavowed any interest in Congress' motive: "the motive and
purpose of a regulation of interstate commerce are matters for the legislative judgment
upon the exercise of which the Constitution places no restrictions and which the Courts
are given no control." Finally, the Court upheld the portion of the Act making it a crime
to employ workers engaged in interstate commerce in violation of the wage/hour
provisions. Given Congress' right to impose direct prohibitions or conditions on
interstate commerce, Congress "may choose the means reasonably adapted to the
attainment of the permitted end, even though they involve control of intrastate activities."
Thus the outright criminalization of employer conduct was a reasonable means of
implementing the prohibition on interstate shipments.
Class notes:
The shipping of goods across state lines may have been a pretext when the real concern
was the wages and labor conditions.
Wickard v.Filburn Pg. 149

CON LAW I Outline Page 36


(Expanding the commerce clause to look at the "cumulative effect" upon interstate
commerce also doing away with the highly formalistic terms such as direct or indirect)
Facts: Wickard involved the Agricultural Adjustment Act of 1938, which permitted the
Secretary of Agriculture to set quotas for the raising of wheat on every farm in the
country. The Act allowed not only the setting of quotas on wheat that would be sold
interstate and intrastate, but also quotas on wheat which would be consumed on the very
farm where it was raised. Filburn, the plaintiff, challenged the government's right to set
a quota on the wheat which was raised and consumed on his own farm, on the grounds
that this was purely local activity beyond the scope of federal control.
Holding: The Court upheld the Act, even as it applies to home-consumed wheat for two
reasons. First, the consumption of home-grown wheat is a large and variable factor in the
economics of the wheat market. The more wheat that is consumed on the farm where it is
grown, the less wheat that is bought in commerce i.e. from other farmers whether
interstate or not. Secondly, the Plaintiff's own effect on the market, by his decision to
consume wheat grown by him, might be trivial. But the decision, "taken together with
the many others similarly situated, is far from trivial. . . ." That is homegrown wheat
supplies a need of the man who grew it which would otherwise be reflected by purchases
in the open market, and the home-grown wheat thus competes with wheat in commerce.
Protection of the interstate commercial trade in wheat clearly falls within the commerce
clause, and the regulation of home-grown wheat is reasonably related to protecting that
commerce.
Class Notes:
One guy alone may not make a difference, but looking at the aggregate impact you will
see that there is a significant effect. This approach looks more at the effect rather than at
the formalistic commercial aspect of the activity.
Heart of Atlanta Motel v. US (1964) pg. 183
F: Title II of Civil Rights Act stated that all persons should be entitled to full and equal
enjoyment of the goods, services, and accommodations of any place of public
accommodation The statute defines a place of public accommodation as those whose
operations affect commerce. A motel challenged this act saying it was unconstitutional.
I: Whether the act was constitutional.
H: Yes
R: Because the disruptive effect that racial discrimination has had on commercial
intercourse is overwhelming.
ROL: Congress has constitutional power under the Commerce Clause to criminalize
racial discrimination in hotels.
Class notes:
Another pretext case? It may be, or this may seriously have an impact on the commerce.
Class Notes
Justified under the commerce clause?
o What is being regulated?
Discrimination/activity (non-economic activity in an economic
context)
Effects Interstate travel inhibited by discrimination; Economic
Development excluding a large % of people from the economy
Isnt it really about 14th amendment?

CON LAW I Outline Page 37


What kind of Effect are we looking at?
o Should it matter that they had pre-text? Gibbons (Marshall) if there is a
pretext, then we should not allow it; Darby pretext doesnt really matter
o May not even have to look at it as a pretext
o Could congress have thought, not what they actually did think

Katzenbach v. Mcclung (1964) pg. 184


F: Companion case to previous case. This case involved a restaurant that did only take
out to blacks.
R: Title II is constitutional because discrimination in restaurants has a direct and highly
restrictive effect upon interstate travel by blacks. This discourages travel and obstructs
interstate commerce.
ROL: Congress has constitutional power under the Commerce Clause to criminalize
racial discrimination in restaurants
Class notes:

The Revival of Limitations on Congresss Commerce Clause Powers


Internal Limits on the scope of the Commerce Clause Power
United States v. Lopez (1995) pg. 186
F: Congress passed the Gun-Free School Zones Act of 1990, which made it a federal
offense to possess a firearm in a school zone.
I: Whether the Gun-Free Act exceeds Congress authority to regulate commerce among
the states.
H: Yes, affirmed.
R: Congress has the power to regulate:
1. Use of channels of interstate commerce
2. Instrumentalities of interstate commerce, or persons or things in interstate
commerce
3. Activities having substantial relation to interstate commerce
The proper test is whether the activity in question substantially affects interstate
commerce. Where economic activity substantially affects interstate commerce, legislation
regulating that activity will be sustained. Here we are dealing with a criminal statute that
has nothing to do with commerce or any sort of economic enterprise. There also does
not seem to be any interstate element here.
The government argues that the issue does affect interstate commerce in the following
ways since violent crimes:
1. Increased insurance costs spread throughout the population
2. Reduced willingness to travel in the country because of the perception that it is
unsafe (like the Heart of Atlanta cases)
3. Threat on the educational process and learning environment.
Also if the court argues against this saying that it is too tenuous to just say this all
happens bc one person has a gun in a school zone, however you could refute this by govt
saying that were not so concerned w/ the one person as we are with the AGGREGATE
effect. The logical implication of this is that it will effect insurance prices for schools,
which is something that families have to deal withso we can argue that this is a family

CON LAW I Outline Page 38


law issue, WHICH IS UNCONST. FOR CONGRESS TO REGULATE B/C IT IS THE
DOMAIN OF THE STATES.
Costs of crime reasoning could regulate all activities that lead to violent crime, no
matter how tenuously related to interstate commerce
National Productivity reasoning could regulate any activity found to relate to the
economic productivity of individual citizens (family law)
Courts response: The response to this is that this is a slippery slope argument and that
if we accept this then any activity can be categorized in a way that the Congress would be
given the authority to regulate it (family law etc.). There must be a stopping point
somewhere. We go by history and tradition to see where states have historically been
involvedthis is a functional argument, not framers intent argument, The possession of a
gun in a local school zone is in no sense an economic activity that through repetition
elsewhere might substantially affect interstate commerce. To uphold the Act as
constitutional would pile inference upon inference in a way that would bid fair to convert
the congressional authority under the commerce clause to a general police power of the
sort retained by the states. Also if just one person, i.e. Lopez, has a gun in schoolthen
there goes the economyis absurd. So it does not affect interstate commerce.

Is this case overruling Wickard case (wheat grower case)? But its not overruled even
though Wickard selling his wheat on the open market wont have any effect on the
economybut it doesnt overrule wickard b/c wickard deals with buying and selling
wickard is selling more wheat than hes allowed to and we can look at the aggregate
effects of this on the economy, which congress may regulate under the commerce clause.
However gun regulation is not in its nature an economic activity.

Courts argument:
a) The economic v. non-economic activities are difficult to distinguish b/c at a point
ask:
1. does it have a substantial effect on an act of commerce
2. if so, is it an economic activity?
3. if not, can you aggregate it and let it go to commerce?

ROL: Gun-Free Act is struck down as unconstitutional.


Kennedy and OConnor concurring: The framers intended that there be a balance of
powers and that the judiciary interpret and enforce that. This law by Congress interferes
and contradicts that balance since there is not a strong connection with commercial
concerns. Kennedy's concurrence suggest that he and O'Connor are less eager than
Rehnquist or Scalia and Thomas to cut back the Court's prior Commerce Clause
interpretations. He said that he had "some pause" about joining the majority opinion,
and that this was a "necessary though limited holding." More than Rehnquist, Kennedy
seemed eager to leave untouched prior cases holding that Congress has full power to
regulate what are truly commercial transactions, even if the transaction being regulated is
a purely local one. But the activity being regulated here, Kennedy sad was not
essentially commercial. Furthermore, Kennedy suggested that activities that had
traditionally been left to the states to regulate should be further off-limits to the federal
commerce power than activities that had not been so limited. Education was one of those

CON LAW I Outline Page 39


traditional concerns of the states. To allow the federal government to interfere would
"foreclose the states from experimenting and exercising their own judgment . . ." in the
area traditionally left to them.
Thomas Concurring: If we allow such a broad construction of commerce to include
this Act, than most of Article I Sec. becomes superfluous and that cannot be so. The
aggregation principle where we look not only at the effects of a single activity on
commerce, but at a whole category of activities, has no stopping point.
Stevens dissenting: Firearms are inherently articles of commerce, and they can be used
to restrain commerce. The Act should be upheld. If you want to prohibit the shipment of
something across state lines, there are other ways to do it besides just place a statutory
limit on it
Souter dissenting: The practice is to defer to rationally based legislative judgment.
Crime and education are certainly areas that affect commerce.
Breyer, Stevens, Souter, and Ginsburg dissenting:
"Rational basis" test: For Breyer, the test was " whether Congress could have had a
rational ba sis for finding a significant or substantial connection between gun-related
school violence and interstate commerce." (The majority had not mentioned "rational
basis" to the majority, the question was whether there actually was a substantial
connection, not merely whether Congress could rationally have believed that there was.)
Crime in schools and education are matters that affect commerce. The only question is
whether the effect is substantial. Deciding that this Act is constitutional does not
expand the meaning of the CC, it only means that this activity has a substantial effect on
commerce.
The statute is aimed at curbing a particularly acute threat to the education
process
The immediacy of the connection between education and the national economic
well-being
Rational Basis Test: a test whereby a court will uphold congressional action if it bears a
reasonable and rational relationship to the attainment of an appropriate government end
The majority opinion has three problems:
1. The Court has held other Acts to be Constitutional for activities that seemingly
have less of an affect on commerce (loan sharking, racial discrimination at
restaurants). Here, the violence will affect where businesses setup shop and where
families choose to live.
2. The majority does not consistently distinguish between commercial and non-
commercial activities in a consistent manner with previous cases.(formal vs.
realistic approach).
3. The majority is basing its opinion (unlike precedent) on a single noncommercial
activity without considering similar instances of the forbidden conduct.
It would be consistent with previous interpretation of the CC to find this activity included
in the type that Congress could regulate.
The statute regulated a noncommercial activity rather than a commercial activity.
The statute regulated area-crime and education- are traditionally subject to state
regulation.
Congress made no findings with respect to the impact of the regulated activity on
the national economy.

CON LAW I Outline Page 40


Class notes:
(There are three broad categories of activities which Congress can constitutionally
regulate)

a) Channels: First, Congress can regulate the use of the "channels" of interstate
commerce. Thus Congress can regulate in a way that is reasonably related to
highways, waterways, and air traffic. See Darby, Heart of Atlanta
b) Instrumentalities: Second, Congress can regulate "instrumentalities" of interstate
commerce, "even though the threat may come only from intrastate activities." This
category refers to people, machines, and other "things" used in carrying out
commerce. So, for instance, presumably Congress could say that every truck must
have a specific safety device. See Perez
c) Substantially affecting commerce: Finally, Congress may regulate those activities
having a "substantial effect" on interstate commerce. See Jones & Laughlin Steel
1) The activity itself must be arguably commercial then it doesn't seem to matter
whether the particular instance of the activity directly affects interstate
commerce, as long as the instance is part of a general class of activities that
collectively substantially effect interstate commerce. Thus Wickard v. Filburn
type of fact pattern i.e. wheat growing activities are in a sense "commercial"
but they are entirely intrastate; however, when taken together with all other
wheat growing we have a substantial effect on interstate commerce.
2) The Court will not give much deference to the fact the Congress believed that
the activity has the requisite "substantial effect" on interstate commerce. The
Court will basically decide the issue for itself. It will no longer be enough that
Congress had a "rational basis" for believing that the requisite effect existed.
If what is being regulated is an activity the regulation of which has traditionally been the
domain of the states, the Court is less likely to find that Congress is acting within its
Commerce Power. Thus education, family law and general criminal law are areas where
the Court is likely to be especially suspicious of congressional "interference."
* The Lopez majority took note that the statute in question contained no jurisdictional
element that would ensure through case by case inquiry, that the firearm possession in
question affects interstate commerce. For example
In U.S.v. Bass the statute there made it a federal crime for a felon to receive, possess, or
transport in commerce or affecting commerce, any firearm.
The statute regulated a noncommercial activity rather than a commercial activity.
The statute regulated area-crime and education- are traditionally subject to state
regulation.
Congress made no findings with respect to the impact of the regulated activity on
the national economy.

Can this holding be reconciled with Civil Rights Cases? Yes, all about kind of activity
possession v. discrimination in sales (non-economic v. economic activity)

How do you determine if it is direct or indirect? Who makes that determination?

US v. Morrison (2000) pg. 197

CON LAW I Outline Page 41


F: Violence Against Women Act of 1994 provides a damage remedy for anyone who
commits a crime of violence motivated by gender. The Court held that this Act was
unconstitutional. The Court decided this using the formalistic approach saying that this is
not of a commercial nature or economic activity. This approach will keep from the
slippery slope argument whereby Congress could draft any law to somehow affect
interstate commerce.
Class notes:
Argument for unconstitutional the activity being regulated is not economic
(violence, discrimination)
o However, in policy terms, discrimination is a federal issue under the 14th
amendment
o But family law and relationship law has been a province of the states since
time immemorial
What about justice Breyers suggestion of passing a workplace violence act that
gives victims of workplace violence damages rather than just victims of
everyones violence?

The 10th Amendment as an External Limit on the Scope of


Commerce Clause
Power CB 154-156, 233-256

The 10th Amendment provides "The powers not delegated to the United
States by the Constitution, nor prohibited by it to the states, are reserved to
the states respectively, or to the people."

In National League of Cities v. Usery (1976) pg. 333 the Supreme Court held that the
10th Amendment barred Congress from making federal minimum wage and overtime
rules applicable to state and municipal employees. The Commerce Clause did not
empower Congress to enforce the minimum wage provision against the states "in areas of
traditional government functions." The 5-4 majority reasoned that the minimum-wage
provision, as applied to state employees, clearly affected commerce. Thus these
wage/hour provisions could unquestionably be constitutionally applied to private
employers, under the Commerce Clause. But when the wage/hour rules were applied to
state employees, they violated the independent requirement, imposed by the 10th
Amendment that "Congress may not exercise power in a fashion that impairs the states
integrity or their ability to function effectively in the federal system." Thus, if the
wage/hour rules were allowed to stand, the majority reasoned, Congress would have the
right to make "fundamental employment decisions" regarding state employees, and "there
would be little left to the states separate and independent existence."
Class Notes:
Uniformity is not a good argument for saying that Congress can impose a minimum wage
on state employees, because the states can deal with the issue and raise the wages on their
own. Also, no one said that there must be uniformity. So what is the argument to justify
Congress regulating State employee wages? We must look at the effect on commerce. If

CON LAW I Outline Page 42


wages are too low there will be more poverty, and people will buy less goods and overall
commerce will be affected. We look at income levels etc.

Garcia v. San Antonio Metropolitan Transit Authority (1985) pg. 234


(Overruling National league of Cities)
1) Issue: The issue in Garcia was whether the minimum-wage and overtime provisions of
the Federal Fair Labor Standard Act (the same statute in the National League of Cities)
should apply to employees of a municipally-owned and operated mass-transit system.
2) Issue: Is the municipal ownership and operation of such a transit system a "traditional
governmental function?
The Majority opinion contended that the 8-year period following National League of
Cities had shown that it was "difficult if not impossible to distinguish between those
functions that are "traditional governmental functions" and those that are not. The
National League of Cities approach inevitably led to judicial subjectivity. "Any rule of
state immunity that looks to the traditional or necessary nature of governmental functions
inevitably invites an non-elected federal judiciary to make decisions about which state
policies it favors and which ones it dislikes."
Class Notes:
The Court says that we will no longer distinguish between activities that are traditionally
a function of the government and those that are not. Congress will be allowed to regulate
minimum wage even for state employees.

New York v. United States (1992) pg. 337


(Congress may not force a state to enact a certain statue or to regulate in a certain
manner)
--was there in this case a potential Garcia problem buried herein?
Facts: Congress enacted the Low-Level Radioactive Waste Policy Amendments Act of
1985. The Act attempted to force each state to make its own arrangement for disposing
(either in-states or out of states) of the low-level radioactive waste generated in that state.
The Act tried to do this with several types of incentives. Most significant was the "take
title" incentive whereby any state which did not arrange for disposal of its waste would
be required to "take title" to the waste upon request by the waste generator, and would be
liable for damages in connection with disposal of this waste. New-York, unlike most
states, made little progress in solving its waste disposal problem, because local residents
of each community where the state proposed to put disposal sites fiercely objected. N.Y
then sued the federal government, arguing that the "take title" provision violated the 10th
Amendment.
Holding: The majority of the Court agreed that the "take title" provision violated the 10th
Amendment: Congress may not "commandeer the legislative processes of the states by
directly compelling them to enact and enforce a federal regulatory program."
O'Connor reasoned that N.Y. was being put to the choice of two "unconstitutionally
coercive regulatory techniques."; it could either choose to regulate on its own by making
arrangements for disposal of waste generated inside the state, or be forced to indemnify
waste-generators against tort damages. Because Congress could not employ either of
these methods alone, it could not escape the problem by giving the state a choice between

CON LAW I Outline Page 43


the two. The fact that the states may have consented is irrelevant they may not consent
to giving Congress powers that the Constitution has not granted them.
Dissents: Three members of the Court (White, joined by Blackmun and Stevens)
dissented. White argued that this was not an instance where Congress was forcing its
will on the states rather he argued the states had requested this statute. First,
Congress may condition the receipt of federal funds on a state's solving the problem
(provided that the funds in question have something to do with the problem). Second,
Congress could directly regulate the conduct in question, and could therefore take the less
drastic step of telling the states that this direct regulation will follow if the states do not
take care of the problems themselves.
Class Notes:
The problem here is that the Constitution simply does not give Congress the authority to
require the States to regulate. The Constitution instead gives Congress the authority to
regulate matters directly and to pre-empt contrary state regulation. Congress may not
conscript state governments as its agents. There would have been no problem if Congress
enforced the Act directly itself.
14th Amendment says that Congress establishes who is a citizen and who is a resident of a
state. So the states cannot even determine who their citizens are. So we see that the
framers are not just the people back in 1787.
(We look at the 3 techniques to try and construe the Constitution: Text, Intent, and
Function). The txt argument of the 10th Amendment doesnt really get you anywhere here.
Under function, there is confusion in terms of who to blame and get mad at if Congress is
imposing the Act and the State is enforcing it. In a democracy accountability is important.
Reconcile with Garcia: This case does NOT overrule Garcia. Garcia did not single out
state employees it included private employees as well. This case singled out the state
gov.

Printz v. US (1997) pg. 348


(Congress may not compel a state or local government's executive branch to perform a
function)
Facts: Congress enacted the "Brady Bill" aimed at controlling the flow of guns. As a
temporary five-year measure, the law ordered local law enforcement officials to conduct
background checks on prospective purchasers, until a national computerized system for
doing these checks could be phased in. Printz and Mack, county sheriffs in Montana and
Arizona respectfully, objected to the back-ground check requirement and sued. They
argued that under N.Y. v. U.S. Congress could not force them to conduct background
checks on the federal government's behalf.
Holding: By a 5-4 margin, the Court agreed that the Act could not force local executives
to perform according to federal guidelines. Justice Scalia majority opinion noted that in
N.Y v. U.S., the Court had said that the federal government "may not compel the states to
enact or administer a federal regulatory program." Scalia then concluded that the back-
ground check portion of the Brady Bill violated this prohibition. Scalia rejected the
dissent's distinction between compelling state executive-branch officers to perform
ministerial tasks. Even if no policy making was involved here, this did not prevent
Congress' action from being an intolerable incursion into state sovereignty: "It is an
essential attribute of the states' retained sovereignty that they remain independent and

CON LAW I Outline Page 44


autonomous within their proper sphere of authority. . . . It is no more compatible with this
independence and autonomy that their officers be "dragooned" into administering federal
law, than it would be compatible with the independence and autonomy of the United
States that its officers be impressed into service for the execution of state laws." Scalia
seemed to be relying on the structure of federal government i.e. federalism rather than
relying on the 10th Amendment. He also made a separation of powers argument in
supporting his position. Art II sec.1, states "The executive Power shall be vested in a
President of the United States of America."
Dissents: The main dissent was by Justice Stevens. He first pointed out that the federal
commerce power gave Congress the authority to regulate handguns. He then concluded
that this being so, the "necessary and proper clause" gave Congress the right to
implement its regulation by temporarily requiring local police officers to perform the
ministerial step of identifying persons who should not be entrusted with handguns. This
was especially true, since Congress could have required private citizens to help with such
identification: " The 10th Amendment provides no support for a rule that immunizes local
officials from obligation that might be imposed on ordinary citizens."
Analysis: N.Y v. U.S. and Printz v. U.S. stand for the proposition that Congress may not
(1) force a state to legislate or regulate in a certain way; (2) require state executive-
branch personnel to perform even ministerial functions.
Distinguished from Garcia: Garcia seems to apply mainly to generally applicable federal
lawmaking; that case holds that where Congress passes a generally applicable law like
minimum wage that apply to all or nearly all businesses, the 10th Amendment does not
entitle a state's own operation to an exemption, merely because a state is being regulated
along with all others private entities. But where the federal government tries to force a
state or local officials to perform particular governmental functions, that is not part of a
generally-applicable federal scheme, and is instead directed specifically at the state's
basic exercise of sovereignty: the state's right to carry out the business of government.
The federal government may not use such coercion.
Class Notes:
Here law enforcement agencies are being bossed around as opposed to the state
legislature (NY v. US). This is not being imposed on private individuals. There is a
dignity argument that the federal government shouldnt be bossed around although they
can be shoved aside if Congress does the enforcement of regulation itself.

Eleventh Amendment as a Barrier to Enforcement of Commerce Clause


11th Amendment states "The judicial power of the United States shall not be construed to
extend to any suit in law or equity, commenced or prosecuted against one of the United
States by Citizens of another State, or by Citizens or Subjects of any Foreign State."

Reno v. Condon Pg 251


Upheld the federal Drivers Privacy Protection Act (DPPA) The act regulates the
disclosure of personal information in the records of state motor vehicles departments.

Pg 231 - Eleventh Amendment as a Barrier to Enforcement of Congressional


Regulations Enacted under the Commerce Clause: 11th Amendment states "The judicial

CON LAW I Outline Page 45


power of the United States shall not be construed to extend to any suit in law or equity,
commenced or prosecuted against one of the United States by Citizens of another State,
or by Citizens or Subjects of any Foreign State."
Sovereign immunity suits against sov. unnecessary, frivolous, non frivolous, political
process will take of it. If can, big settlements or judgments could effect state treasury,
enforcement could be issue. Florida has sovereign immunity, they can control lawsuits
and there are caps.

Hans v. Louisiana Supp. (1890)


(Federal Court has no jurisdiction due to 11th amendment, even though, not per the exact
wording of the amendment) 11th amendment was ratified after Chisholm v. Georgia was
decided by the supreme court allowing 2 individuals from south Carolina to sue Georgia
or vice versa, repealing any notion of sovereign immunity enjoyed by the states, however
the controversy is that the wording looks like the 11th amendment prohibits only diversity
claims, where the majority in this case has broadened the interpretation to mean all
claims.
Facts: Citizen of Louisiana brought action in Federal court against state of Louisiana
under subject matter jurisdiction, where the state refused to honor interest in regards to a
bond they issued petitioner, allegedly violating the bond contract between the individual
and state violating Article 1 10 law impairing the obligation of contracts, where the
state diverted the interest funds of the contract to other state needs or accounts.
Holding 11th amendment is expanded to cover citizens of state suing state
maintaining state sovereign immunity.
Dissent in Chisolm congress can limit jurisdiction per article III, up to congress to limit
or expand sovereign immunity.
Majority article III got rid of sovereign immunity.
Interpretations (1) Article III eliminates or doesnt adopt state sovereign immunity in fed
court (2) it is up to congress; (3) Article III requires sovereign immunity (not going to get
out of amendment, unthinkable that states could be sued in fed court.) argue with states
consent, frames liked sovereign immunity.
CLASS NOTES

Ex parte Young Supp. (1908)


Minnesota legislature passed a number of statutes regulating railroad rates. Before the
acts took effect, shareholders sought to challenge them as an unconstitutional taking of
property in violation of the 14th amendment. The complaint named the Attorney General
of Minnesota as one of the s. The supreme court maintained the injunction against
enforcing the rate regulation by the Attorney General based on Federal question
regarding the 14th amendment. Narrow holding - Injunction judgment was against an
officer of the state for injunction of enforcement of a State act prohibited by federal law
only. No monetary damages etc, and not against the state. The violations of the state act
were stiff penalties and jail time depending on the number times violating (unreasonable).
Harlans Dissent state are required to enforce fed laws if not can then can petition
the supreme court. Supreme court always has appellate jurisdiction. majority is
infringing on the states sovereign immunity in which it should let the state adjudicate its

CON LAW I Outline Page 46


own. He says that the correct redress is in state court then to the supreme court upon a
writ of error. He also says - parties cannot, in any case, obtain redress by a suit against
the state
Court was looking at Attorney General as working as an individual breaking the federal
law, same as going out and killing someone. However if looking at as individual
(viewing in lieu of 11 amendment, for jurisdiction), then the scope may fall outside 14th
amendment as 14 amendment pertains to states. Another way to look at, 14 occurred
after 11th should take precedence.

Seminole Tribe of Florida v. Florida Supp.


(Congress may not abrogate the 11th Amendment outside of suits arising under the 14th
Amendment)

Facts: Congress passed a statute, the Indian Gaming Regulatory Act, to govern aspects of
gambling operations run by Indian tribes. The IGRA provided that when a state allows
non-Indian gambling, the state must also negotiate in good faith with any tribe located in
the state to try to reach an agreement permitting the tribe to conduct comparable
gambling operations. The statute also provided that if a tribe believed that the state was
not negotiating in good faith, the tribe could sue the state in federal court for an order
directing the state to negotiate in good faith.

Holding: By a 5-4 vote, the Court held the Act violated the 11th Amendment. The
majority opinion, by Justice Rehnquist, held that "even when the Constitution vests in
Congress complete law-making authority over a particular area, the 11th Amendment
restricts the judicial power under Art III, and Art I cannot be used to circumvent the
constitutional limitations placed upon federal jurisdiction." So even though Art. I gives
Congress full authority to "regulate commerce with the Indian tribes, "Congress cannot
allow a tribe to sue a state in federal court. The 11th Amendment embodies concepts of
state sovereignty. That Amendment "reflects the fundamental principle of sovereign
immunity that limits the grant of judicial authority in Art. III. Even though Seminole
itself involved congressional power based on the Indian Commerce Clause, the decision
applies broadly to all sources of congressional power other than the 14th Amendment. For
instance, it clearly applies to congressional action based on the "regular" Commerce
Clause. The Seminole Court held that Congress no longer has the power under the
Commerce Clause to abrogate the states sovereign immunity overruling Pennsylvania v.
Union Gas Co. The majority also rejected the argument proposed by the Seminole Tribe
that the doctrine of Ex Parte Young should apply. That doctrine which provides federal
jurisdiction over a suit against a state official when that suit seeks only prospective
injunctive relief in order to "end a continuing violation of federal law. Here the
continuing violation of federal law alleged is the Governor's failure to bring the state in
compliance with the statute. The Court reasoned that "Where Congress has prescribed a
detailed remedial scheme for the enforcement against a state of a statutorily created right,
a court should hesitate before casting aside those limitations and permitting an action a
state official based upon Ex Parte Young." Like Ex Parte a state official will have to
negotiate, if Seminoles are looking for an enjoinment, requiring good faith negotiations.

CON LAW I Outline Page 47


Congress wasnt that serious about having to negotiate no penalty just in the end, the
secretary of interior comes up with compact.
Seminal thought they could sue Florida as statute stated so.

Dissents: Souter argued that all the drafters of the 11th Amendment ever meant to do and
all the Amendment had ever been interpreted as doing, was to prevent suits based solely
on diversity, i.e. those brought by a state against a non-citizen in circumstances under
which no federal question was raised.
Stevens argued that the majority holding prevents Congress from providing a federal
forum for a broad range of actions states."

State Sovereign Immunity (SI)


Did the 14th Amendment (1868) override the 11th Amendment (1798)? The 14th focused
on individual rights being protected from the States.
Continuation of Seminole case
Why couldnt you use Ex Parte Young in Seminole?
What made them think in this case that they could haul the state of Florida into court?
There may be a scheme where the federal government brings the suit on behalf of the
people. The state of law at the time of this case was somewhat unsettled (in terms of
abrogating SI) plus the Seminoles figured that the feds could get involved pursuant to
Congress Commerce Clause power. Hans would not prevent this suit because unlike
Hans Congress had abrogated SI under the 11th Amendment.
Holding: Congress cannot abrogate the 11th Amendment immunity except when it
does so pursuant to the 14th Amendment.
As other remedies: you may sue in state court (with consent).
Dissent:
Stevens: Congress did not have to grant the full jurisdiction to federal or state courts
under Article III. Chisolm eliminates SI and Article III does not eliminate State SI either.
The 11th Amendment was only intended to make sure that it is up to Congress to decide
when state SI should apply. Congress should control state SI here. Chisolm was wrong in
assuming that states are automatically suable in a federal court it is not all or nothing.
However, why would they use an Amendment to resolve this and what is the language
about citizens in another state. Stevens is no worse than the majority in terms of
explaining the language.
Souter: The 11th Amendment does not grant SI, rather it ensures that there be diversity
and federal question to get into federal court. Souter says that the 11th Amendment came
to limit citizen state diversity. Souter thinks that Hans was not decided correctly. 11th
applies only where the basis of jurisdiction is where the citizen is not from the state who
he is suing. If they wanted to bar all suits by individuals against states (no matter which
state you are from) why wouldnt they have adopted that approach? We want to keep
cases out of federal courts where the only basis is diversity of citizenship without a
federal question. Correctly applied, Hans would protect the state against suit if the sole
basis of jurisdiction was diversity. In Hans they were suing on a contracts issue and not a
federal claim. Hans should have said this is a federal jurisdiction case and therefore the
11th does not apply. Congress ought to control SI and the 11th did not intend to change
that. The 11th was not intended to apply to federal question claims. SI is only intended to

CON LAW I Outline Page 48


apply where Congress leaves it in place (as it did in Seminole). The issue of whether SI is
a good thing depends on how you view the affront to the States dignity.
Alden v. Maine (1999) pg. 50 supp.
Congress cannot abrogate State SI per the 11th Amendment.

Federal Maritime Commission v. South Carolina State Ports Authority (2002) pg. 51
supp.
F: The FMC brought a suit against the state for a violation of the Shipping Act of 1984
(which forbids marine terminal operators to discriminate against terminal users) for
refusal to provide berthing space to a ship as part of an anti-gambling discriminatory
policy while it did provide berthing for two Carnival ships with gambling activities. FMC
held that SI did not bar a suit against the state by the FMC and the USCA reversed and
this Court affirms.
I: Whether state sovereign immunity precludes an administrative agency from bringing a
suit against a state.
H: yes, affirmed.
Reasoning:
The 11th Amendment does not define the scope of the States SI; it is but one
particular exemplification of that immunity. It was a reaction to Chisholm, which
is why it only states that a citizen from another state may not bring an action
against a state.
The text refers to judicial power the framers could not have anticipated the
vast growth of the administrative state. There is a strong resemblance between a
private suit and an administrative proceeding. The same affront to the States
dignity exists in front of an administrative tribunal.
FMCs Arguments:
1. SI should not apply because the FMCs orders are not self-executing the
FMC cannot enforce its own orders. COUNTER: The US does not
exercise political responsibility for complaints, but instead has effected a
broad delegation to private persons to sue non-consenting states.???
2. FMC proceedings do not present the same threat to the financial integrity
of the States as do private suits. COUNTER: The purpose of SI is to
accord the State the respect owed them. Immunity applies regardless of
whether a private P suit is for monetary damages or some other type of
relief. SI is an immunity from suit, not just monetary liability.
3. SI should not apply because of the necessity of uniformity in the
regulation of maritime commerce. COUNTER: The States SI extends to
cases concerning maritime commerce. The US can make sure that the state
complies with the Shipping Act and other Federal rules though.
4. Even if SI applies, other forms of relief such as cease and desist order
should be allowed. COUNTER: SI does not only protect State treasuries,
but also sovereign dignity. The relief sought by a P is irrelevant.
ROL: State sovereign immunity precludes an administrative agency from bringing a suit
against a state.
Stevens dissent:

CON LAW I Outline Page 49


Legislative history and precedent shows that the purpose of the 11th Amendment was
NOT to protect a States dignity. The Amendment overruled Chisholms SMJ holding,
thereby restricting the federal courts diversity jurisdiction and left intact the PJ holding
(that the Constitution does not immunize States from a federal courts process). If the
intention of the 11th Amendment was to protect the States dignity, Congress would have
endorsed the first proposal of the amendment rather than the later which merely
delineates the SMJ of the courts.
Breyer, Stevens, Souter, Ginsburg dissenting:
The SI that the majority is applying here is nowhere to be found in the text of the
constitution. This is an administrative agency independent from the three branches of
govt. If anything, it is part of the executive branch. Here we have a typical Executive
branch agency exercising typical powers seeking to determine whether a federal law has
been violated. The constitution created a federal govt empowered to enact laws that
would bind the States. Federal administrative agencies do not exercise judicial power.
Administrative state where an individual can complain to the agency and the agency
through the federal govt may sue the State. There is no affront to the dignity of the
state. Why should there be a special constitutional protection? The majority decision
allows the agency to bring enforcement actions against states, however it forbids using
agency adjudication. The result is less flexibility and a larger federal bureaucracy.
Class notes:
Majority says that if what is going on in the agency looks like a lawsuit, then we will
treat it as one and apply State SI. Admin. agencies are not even mentioned in the
Constitution.

Others Powers of Congress


The Treaty and War Powers
Assume we go back to Usery where it was held that Congress, under the Commerce
Clause, could not interfere with state sovereignty to compel states to pay minimum wage
to employees. Now if there were a treaty compelling a state to pay overtime what would
happen?

Missouri v. Holland (1920) pg. 203 (Congress can constitutionally enact a statute under
Art I sec. 8 to enforce a treaty created under Art II sec. 2, even if the statute by itself is
unconstitutional)
Art II sec.2 cl.2 states in pertinent part "He shall have Power, by and with the Advice and
Consent of the Senate, to make Treaties, provided two thirds of the Senators present
concur. . . ."
Art VI provides in pertinent part "This Constitution, and the Laws of the United States
which shall be made in Pursuance thereof; and all Treaties made or which shall be made,
under the Authority of the United States, shall be the supreme Law of the Land . . ..
Art I sec. 8 cl. 18 states the Congress shall have the power "To make all Laws which shall
be necessary and proper for carrying into Execution the foregoing Powers, and all other
Powers vested by this Constitution . . . ."
Facts: The U.S. entered into a treaty with the United Kingdom to protect birds that
migrated between Canada and the U.S. Congress passes a statute to enforce the treaty.
Missouri filed a bill in equity to prevent Holland, the U.S. game warden, from enforcing

CON LAW I Outline Page 50


the treaty. Missouri claimed that the statute unconstitutionally interfered with the rights
reserved to the states by the 10th Amendment.
Issue: Whether Congress can validly enact a statute to enforce a treaty if the statute
standing alone would be unconstitutional because it violates the 10th Amendment.
Holding: Yes, Art II sec. 2 grants the president the power to make treaties and Art VI sec.
2 declares that treaties shall be part of the supreme law of the land. "The only question is
whether it is forbidden by the same invisible radiation from the general terms of the 10th
Amendment." Therefore, if a treaty is a valid one, Art I sec. 8 gives Congress the power
to enact legislation that is a "necessary and proper" means to enforce the legislation. " No
agreement with a foreign nation can confer power on the Congress, or on any other
branch of government, which is free from the restraints of the Constitution. . . ." The
Court held that the state of Missouri did not have pecuniary interest as owners when the
birds were in the state. There is no ownership until there is possession, and the birds
never came into possession. See Professor Abraham's 4th lecture entitled "Possession in
Wild Animals."
Class Notes:
Does Reid on pg. 205 (bottom) overrule Holland? In Reid the treaty was invalid and in
Holland it was ok. They can be reconciled to say that the rule is that a statute is valid as
long as it is consistent with other provisions of the constitution. In Holland there was no
inconsistency between the treaty and the constitution. However in Reid the treaty violated
the Bill of Rights and constitutional history.
A statute must be consistent with the constitution, whereas a treaty must only be made in
pursuant of the constitution. Treaties dont really have to be subject to constitutional
strictures. In Reid the treaty conflicted with a specific provision of the constitution and
therefore it was invalid, however in Holland the treaty did not directly conflict with a
constitutional provision.

War Power pg. 206


Woods (1948) question of constitutionality of the Housing and Rent Act related to
issues resulting from the war. The SC upheld the constitutionality of this statute. Here we
see the war power extending further than the commerce clause power did (Lopez).

The Spending Powers


Art I sec. 8, gives Congress the power "to lay and collect Taxes. . . to pay the Debt and
provide for the common Defense and general Welfare of the United States. . . ." The
power to spend is thus linked to the power to tax money may be raised by taxation, and
then spent "for the common Defense and general Welfare of the United States."

U.S. v. Butler (1936) pg. 209 "The power to confer and withhold an unlimited benefit is
the power to coerce or destroy"
Holding: Congress has no power to regulate for the purpose of providing for the "general
welfare. Congress may spend for the general welfare, it may tax for the general welfare,
but it may not regulate for the general welfare. Thus, a congressional regulatory scheme
has to be justified as a reasonable means of carrying out some other enumerated power,
this occurs mostly in the area of interstate commerce)

CON LAW I Outline Page 51


Facts: Butler involved the validity of the Agricultural Adjustment Act of 1933, a New
Deal measure sought to raise farm prices by cutting back agricultural production. The
scheme was to be carried out by authorizing the Secretary of Agriculture to contract with
farmers to reduce their acreage under cultivation in return for benefit payments; the
payments were in turn made from a fund generated by the imposition of a "processing
tax" on the processing of the commodity.
Arguments on framers intent: Madison asserted the taxing and spending clause
amounted to no more than a reference to the other powers enumerated in the subsequent
clauses i.e. section 8 of the same section; that, as the United States is a government of
limited and enumerated powers, the grant of power to tax and spend for the general
welfare must be confined to the enumerated legislative fields committed to Congress.
Hamilton, on the other hand, maintained the clause confers a power separate and
distinct from those later enumerated, and Congress consequently has a substantive
power to tax and appropriate limited only by the requirement that the tax is for the
general welfare.
Holding: The Court first concluded that the power to "tax and spend for the general
welfare" existed as a power separate and distinct from the other powers enumerated in
Art I sec. 8, adopting Hamiltons approach. Thus the taxing and spending power stood on
equal footing with, for example, the power to regulate interstate commerce. By this
standard, there was no difficulty with the Agriculture Adjustment Act. The Court,
however, rejected the contention that Congress had an independent power to "provide for
the general welfare" apart from the power to tax and spend. Therefore Congress may
not regulate in a particular area merely on the grounds that it is thereby providing
for the general welfare; Congresss taxing and spending may, however, be done for
"the general welfare." Otherwise the federal government would be one of a "general
and unlimited powers" rather than enumerated and limited one. Therefore, Congress had
no right to regulate areas of essentially local control, including agriculture. Because
Congress could not directly regulate agriculture, it also could not coercively purchase
compliance with a regulatory scheme. Additionally, the Court distinguished the Act from
a conditional appropriation of money (which would be valid, even under the Court's
view.) What was impermissible in the Act was the fact that the farmer contractually
binds to obey the regulations which regulations he could not be directly commanded to
obey. The use of contracts in this way, "would tend to nullify all constitutional
limitations upon legislative power."
Dissent: The power to tax and spend includes the power to relieve a nationwide economic
maladjustment by conditional gifts of money.
Class Notes:
What is or isnt voluntary? The court is saying that it is voluntary cant purchase
compliance, therefore it is not a regulation. On the other hand it is not voluntary and this
is a regulation that Congress cannot do.
One may argue that the situation in Butler was more coercive because of their poor
economic situations. Voluntary vs. coercion. They say that they were offering such an
economic incentive that it constituted coercion. Even if something is voluntary, should
the government be able to hand out money with strings attached?

Dole pg. 217 (1987)

CON LAW I Outline Page 52


Government gives financial incentives for states to build highways as long as they make
the legal drinking age 21. Is this for the general welfare and who will decide if it is or
isnt? Dole said that Congress would decide if it is for the general welfare. There must
also be a connection between what the government wants the state to do in return
for the incentive. (drinking and driving and roads). The string attached must be
explicit so that the states know what they are getting into.

-federal economic might bears on States own choice of public policy. public
spending into unearned private gain.and thats ok here, but in Sabri the issue is different
b/c bribery imposes criminal statute as authority to bring federal power to bear directly
on INDIVIDUALS who convert

Bailey v. Drexel Furniture Company, 1922 (pg. 281)


The Supreme Court struck down the Child Labor Act, which actually set age limits on
workers and therefore set out to protect workers (might look like the Court is corrupt),
saying it is a tax as a penalty and so is NOT proper under Congress taxing and spending
powers.
R: because the purposes of the Act are palpable and the act demonstrated its purpose
by setting forth a detailed and specified course of conduct in business. So this was
not incidental restraint that is normal of any sort of tax; rather, it was clearly designed to
stop the conditions in the act, to protect workers, and while this is a good thing, it is not
within Congress taxing and spending power to enforce.
So-called taxing act must be naturally and reasonably adapted to the collection of
the tax and not solely to the achievement of some other purpose plainly within state
power.

United States v. Doremus (pg 282)


The court refused to declare an act of congress, which imposes a tax, invalid.
R: because the taxing act was naturally and reasonably adapted to the collection of a tax
and nto solely to the achievement of some other purpose plainly within state power.

HYPO HIGHWAY PARTISANSHIP


1. exercise of spending power must be in pursuit of the general welfare
2. condition must be done so unambiguously
3. condition must be related to the federal interest in particular national programs or
projects
could argue that one would not want patronage, fear of corruption
may use a slide-scale if there is a strong connection!

Sabri v. United States (2004) pg 292


(upheld the constitutionality of a federal statute, making it crime to bribe a state or local
official of an entity that receives at least $10,000 in federal funds, WITHOUT showing
any connection between the bribe and the funds.)

CON LAW I Outline Page 53


Discussed Dole in these terms (distinguishing the two), the criminal statute is authority
to bring federal power to bear directly on individuals

Steward Machine Co. v. Davis (1937) pg. 215


(a federal unemployment insurance tax statute which provides a credit for contribution
made to state unemployment funds is valid since (1) Congress enacted it primarily to
safeguard its own treasury and; (2) the proceeds of the tax are not earmarked for a special
group.
Facts: The Social Security Act imposes an excise tax on companies that employ eight or
more employees based on the wages paid. The proceeds go into a general treasury,
however, if a taxpayer has made contributions to a state unemployment fund such
contributions may be credited against a federal tax, so long as the state law meets the
social security board standard. Between 1929-1936 there was an unprecedented number
of unemployed people and the states were unable to give the requisite relief needed.
Steward Machine Co. paid a tax in accordance with the statute and filed this claim to
recover payment asserting that the statute is unconstitutional.
Issue: Is a federal unemployment insurance tax which provides for credit for
contributions into a state unemployment fund constitutional?
Holding: yes, under Art I sec 8 cl. 1 "Congress shall have the power to
collect taxes to promote the general welfare." Also, based on the extreme situation in
unemployment the scheme was essential to aid in the general welfare.
Notes 242-248
The Court distinguished Butler on four grounds: 1) the money here was not earmarked
for a special group and was instead going to the general welfare; 2) the system operated
in a state only if the state gave its approval and; 3) the state could repeal its law at its
pleasure; 4) the end, unemployment relief was one on which the nation and state may
lawfully cooperate i.e. dual federalism. (There is a difference between the threat to
take away and to gain).

The modern view: South Dakota v. Dole (providing the black letter law of Congress
spending power)
Facts: Congress, in order, to prevent drivers under the age of 21 from drinking withholds
federal highway funds from states that permit individuals younger than 21 to purchase to
purchase or possess in public any alcoholic beverage. South Dakota attacks the statute on
the grounds that this condition interferes with its own exclusive powers under both the
10th and 21st Amendemts.
Holding: The statute was upheld the spending power is not unlimited but instead subject
to 4 restrictions: (1) the spending power must be in pursuit of the general welfare, Courts
usually allow Congress much discretion in this area, (2) if Congress decides to condition
the receipt of federal funds it must do so unambiguously , (3) the conditions on the
federal grant must be related to the federal interests, (4) finally other constitutional
provisions may provide an independent bar to the conditional grant of federal funds.
Here the 21 year old drinking age satisfied each of these requirements. First, it served the
general welfare because different drinking ages in different states create incentives for
young people to combine their desire to drink with their ability to drive and thus
reasonably connected to serve the general welfare. Secondly, the conditions imposed by

CON LAW I Outline Page 54


Congress is directly related to one of the main purposes for which highway funds are
expended i.e. safe interstate travel. Finally, the Court discussed the question of coercion.
It agreed with the suggestion in Steward Machine Co. that at some point pressure turns
into coercion. But here because all South Dakota would lose was only a small portion
of the highway funds. 5%
Class Notes:
Level of coerciveness.

Elderman v. Jordan supp. Pg. 65


(Any action in a federal court to award retroactive benefits requires waiver by the state
of its 11th Amendment sovereign immunity)
Facts: Jordan, the plaintiff, filed a complaint, individually and as a representative of a
class, seeking injunctive relief against two former directors of the Illinois Department of
Public Aid. Jordan claimed that the state officials were administering a federal-state
program of Aid to the aged, blind or disabled, in a manner inconsistent with various
federal regulations and the 14th Amendment. Such actions of the state officials were
alleged to violate federal law & deny equal protection under the laws. Plaintiff's prayer
for relief requested declaratory & injunctive relief, and a specifically requested a
permanent injunction directing the defendants to award to the entire class of plaintiffs all
(not future) the (AABD) benefits wrongfully denied.
Issue: Whether the granting of retroactive benefits, in an action by a citizen against a
state in a federal court is in violation of the 11th Amendment.
Holding: Yes, although Ex Parte Young does not bar that part of the district court's ruling
that prospectively enjoined the defendant from failing to process applications with in the
time allowed by federal rules, it does not extend so far as to permit a suit which seeks the
award of an accrued monetary liability which must be met form the general revenues of a
state, absent consent or waiver by the state of its 11th Amendment immunity. Therefore
the award of retroactive benefits was improper. Even though Ex Parte Young granting
future injunctive relief had an effect on the state treasury, as a necessary result of
compliance in the future with the federal rule, it was an unavoidable consequence. But
such an ancillary effect on the state treasury is a permissible an often inevitable
consequence. The difference here is it requires payment of state funds not as a necessary
consequence of compliance in the future, but as a form of "compensation to those who
suffered when the state was under no court-imposed obligation to conform to a different
standard.
Class Notes:
Injunctive relief is ok prospective, but not retrospective. States can waive their 11th
Amendment immunity. The waiver must be very explicit and not constructive.

Kimel v. Florida Board of Regents (2000) supp. Pg. 67


(Professors Schnably summary of the 11th Amendment)
Facts: The Age Discrimination in Employment Act of 1967 makes it unlawful for an
employer, including a state, to fail or refuse to hire or to discharge any individual or
otherwise discriminate against individuals because of the age. A group of current and
former faculty and librarians of Florida State University filed suit against the Florida
Board of Regents in the United States District Court.

CON LAW I Outline Page 55


Issue: First, whether Congress unequivocally expressed its intent to abrogate that
immunity; and second, if it did, whether Congress acted pursuant to a valid of
constitutional authority.
Holding: It is clear that Congress intended to abrogate the states sovereign immunity so
the second issue is critical to our analysis.
The ADEA is not a valid exercise of Congress power under section 5 of the 14th to
abrogate state SI in this case.
In EEOC v. Wyoming the Court found the ADEA constituted a valid exercise of
Congress power to regulate commerce . . . among the several states, Art I sec 8, cl. 3
and the Act did not did not transgress any external restraints imposed on the commerce
power by the 10th Amendment. Because the Act was found valid under the commerce
clause, the Court never considered whether the act was valid under section 5 of the 14th
Amendment. However, because Congress cannot abrogate the states 11th Amendment
sovereign immunity based on the Commerce Clause the only way Congress can abrogate
a states sovereign immunity is through section 5 of the 14th Amendment. Remember!
The Seminole Court held that Congress no longer has the power under the Commerce
Clause to abrogate the states sovereign immunity overruling Pennsylvania v. Union Gas
Co.
Section 5 of the 14th Amendment grants Congress the authority to abrogate the States
Soverign Immunity: In determining if sec 5 is used correctly in abrogating a states
sovereign immunity the Supreme Court laid down the congruence and proportionality
test. Applying that test to the present case the statute here may not abrogate the
states sovereign immunity. Our examination of the ADEAs legislative record confirms
that Congress 1974 extension of the Act to the states was an unwarranted response to a
perhaps inconsequential problem. Congress never identified any pattern of age
discrimination by the states, much less any discrimination whatsoever that rose to the
level of constitutional violation. But relied on newspaper articles and private practices.
A review of the ADEAs legislative record as a whole, then, reveals that Congress had
virtually no reason to believe that state and local governments were unconstitutionally
discriminating against their employees on the basis of age. Although the lack of support
is not determinative of the section 5 inquiry, Congress failure to uncover any significant
pattern of unconstitutional discrimination here confirms that Congress had no reason to
believe that broad prophylactic legislation was necessary in this field. Thus, Congress
was not attempting to remedy a pattern of state discriminatory practices.
Notes:
*Professor Schnably was actually correct in my disagreement over whether a legislative
record was pertinent to an inquiry in the abrogation of a states sovereign immunity based
on section 5 of the 14th Amendment.
Summary of the 11th Amendment States Sovereign Immunity
An individual can get around a states sovereign immunity by applying the Ex Parte
Young doctrine that allows an injunction against a state official who is presently violating
the law. An injunction, however, may only be for prospective injunctive relief. See
Elderman v. Jordan. A federal statute may not abrogate a state sovereign immunity based
on its Commerce Clause powers. See Seminole Tribe
A federal statute may abrogate a state sovereign immunity based on sec 5 of the 14th
Amendment only if there is a clear showing that the statute enacted was intended to

CON LAW I Outline Page 56


remedy a state sanctioned discriminatory practice. See Kimel v. Florida Board of
Regents
The federal government may bring its own action against a state.
Class Notes:
Congress was going after age discrimination. States may discriminate on the basis of age
if it is rationally related to a legitimate state interest. The other side would say that
intentional discrimination is very hard to prove, and that discrimination is widespread.
This was not just an exercise of Commerce Clause, but it is also sec. 5 of the 14th. The
Court is saying that they do not see a national widespread discrimination problem.

11th Amendment: To sue the State in a federal court, there must be:
1. Consent
2. Congress abrogates the immunity (properly applying the 14th)
3. You sue a state official and not the state (for prospective only, not retroactive)

Souters interpretation of the 11th as an alternative: 11th was not intended to apply to
federal question jurisdictions, but only to diversity of citizenship.
Stevens: States are shielded from suit in federal court unless Congress says otherwise.

The Power to Enforce the Reconstruction Amendments


Katzenbach v. Morgan (1966) pg. 222
F: NY enacted a law saying that Puerto Ricans with a certain level of education could
not vote. The federal govt under sec. 5 of the 14th rendered that unconstitutional on the
grounds of discrimination.
R: Congress had the authority to say that the need of the PR minority to vote warranted
federal intrusion upon any state interests served by the English literacy requirement.
ROL: Federal intervention is constitutional.
Dissent: section 5 is not being applied appropriately here.
Class notes:
Is Congress disagreeing with the SC, because in an earlier case (1959) the Court held that
the NY statute of giving literacy tests was held constitutional? And here (6 years later)
Congress passes legislation saying that giving literacy tests as a prerequisite to voting is a
violation of Equal Protection. The 14th is different because of section 5, which says that
Congress shall have power to enforce, by appropriate legislation, the provisions of this
article. Is section 5 of the 14th a modification of Marbury v. Madison? It may be that this
section gives final authority to Congress when it comes to the 14th. One way to interpret
this case is as being remedial (remedy) that if we can allow these people to vote they can
later right the wrongs of discrimination in housing and welfare. In other words the
discrimination in housing and welfare is the real wrong, and the literacy test and voting is
really just a means to an ends. There is a second part saying that this decision is also
substantive that a literacy test violates the 14th.
Ratchet Theory that Congress can be more protective but not less.
Congress can disagree with interpretation of Constitution

CON LAW I Outline Page 57


City of Boerne v. Flores (1997) pg. 226
Facts: A decision delivered by local zoning authorities to deny a church a building permit
was challenged under the Religious Freedom Restoration Act of 1993 (RFRA). This case
calls into question the authority of Congress to enact RFRA. RFRA prohibits
government from substantially burdening a persons exercise of religion even if the
burden results from a rule of general applicability unless the government can
demonstrate (1) the burden is in furtherance of a compelling interest and (2) is the least
restrictive means of furthering that compelling governmental interest. The Acts mandate
applies to any branch, department, agency, instrumentality, . . .as well as to any state or
subdivision of a state. In essence the RFRA Act shifted the burden to the state to show a
compelling justification for a facially neutral statute.
Issue: Is section 5 of the 14th Amendment a grant of congressional authority to remedy as
well a prevent discrimination.
Holding: No. Sec 5 of the 14th Amendment allows Congress to enact statutes that
remedy discrimination not prevent it.
If Congress could define its own powers by altering the 14th Amendments meaning, no
longer would the Constitution be superior paramount law, unchangeable by ordinary
means. It would be on a level with ordinary legislative acts and like other acts alterable
when the legislature shall please to alter it. See Marbury v. Madison
Class Notes:
For Congress to be able to intervene, the locality would have to discriminate and single
out religions (1st Amendment issue). The court says that the test must be one of
congruence and proportionality. To deal with the problem (discrimination) Congress
wants to put forth a remedy that casts a much wider net than the actual problem.

Analysis
Under Section 5 of the 14th amendment, Congress has the power to enact legislation
to enforce the guarantees of equal protection and due process
In giving a limited reading of Section 5, the Supreme Court not only limits Congress
power to enforce by legislation the 14th, but also expands its own power
The court states that Congress enforcement power is just that power to enforce
It has no right to determine what actions constitute a constitutional violation, but only
the power to provide remedies for those violations already enumerated
If Congress cannot determine what constitutes a constitutional violation, then
who can?
The Judiciary by deciding Boerne as it did, the SC protected its power to
determine constitutional violations from encroachment by other branches of
government

RFRA congressional act designed to prohibit the government from burdening a


persons religious practices even to the extent that those practices conflicted with laws of
general applicability unless the government can show a compelling interest in enacting
the law

CLASS NOTES
Difficult to prove intent/motivation, easier to shower harm, or effect of burdening!

CON LAW I Outline Page 58


IV. Constitutional Limits on State Powers (Dormant Commerce


Clause)
A. Introduction
Failure of Articles of Confederation: The pre-Constitution Articles of
Confederation had failed largely because states fought destructive trade wars
against each other. These trade wars arose principally from the fact that state
government were too responsive to local economic interest i.e. each state
government tended to pursue the interests of its own constituents, at the expense
of citizens of other states. Therefore, under the Constitution, the power of the
federal judiciary, interpreting the Commerce Clause, had to prevent this
economic balkanization.

Dormant Commerce Clause: the mere existence of the federal commerce power
restricts the states from discriminating against, or unduly burdening, interstate
commerce. This restriction is referred to as the Negative Commerce Clause or the
Dormant Commerce Clause.
Art I sec 10 expressly disables states from certain activities that are elsewhere
committed to Congress, and it does not expressly prohibit the States from
regulating interstate commerce (for example the power to declare war is clearly an
exclusive power of Congress). Why should the commerce power standing alone
be exclusive?

A problem with the exclusive approach is that almost everything will have an
effect on interstate commerce (IC) so that you would completely hamstring the
power of the states if you define it that way. The Courts have not taken the
exclusivity approach of the Commerce Clause. The general acceptance today is
that both Congress and the States can regulate interstate commerce. We will look
at when the states can and cannot. Rule is that the state cannot regulate if they are
facially discriminating against other states, otherwise a balancing test is applied. It
is very tough to tell whether something is facially discriminatory or not.

Early interpretations:

Direct versus indirect


In Disanto v. Pennsylvania pg. 258 the Court invalidated a licensing statute for those
wishing to sell tickets for transportation to or from a foreign country. The state argued
that the statute was designed to prevent exploitation. The majority was not persuaded by
the states arguments noting that Congress has complete and paramount authority to
regulate foreign countries a statute which by its necessary operations directly interferes
with or burdens foreign commerce is a prohibited regulation and invalid regardless of its
purpose.

CON LAW I Outline Page 59


Inherently local versus national
Cooley v. Board of Port Wardens
Facts: A Pennsylvania law was adopted requiring all ships entering or leaving the port of
Philadelphia to use a local pilot or pay a fine to support retired pilots and their
dependents. The Court agreed that the regulations of pilots was a regulation of interstate
commerce, even though pilots stayed with the ships for only a brief periods but it
nevertheless upheld the statute. The Court supported this conclusion by referring to a
federal statute providing that all pilots shall be regulated in conformity with such laws as
the states may require. The Court also took notice of the distinction between a regulation
requiring diversity and a regulation requiring uniformity stating in pertinent part
The power to regulate commerce, embraces a vast field, containing not only
many by exceedingly various subjects, quite unlike in their nature; some imperatively
demanding single uniform rule, operating equally on the commerce of the U.S. in every
port; and some, like the subject now in question, as imperatively demanding that
diversity, which alone can meet the local necessities of navigation. The Cooley
reasoning requires that a Court determine whether a subject is of a nature requiring
uniform national regulations or diverse local regulations.
The question is whether Congresss power under the Commerce Clause is exclusive. The
dormant Clause means that even if Congress is silent by not regulating something, does
that mean that the States may not regulate. Scalia argued that there is no indication of
exclusivity and that if the CC were exclusive the Constitution would have said so as it did
regarding the power to coin money or make treaties or declare war.

The Prohibition of Facial Discrimination against Interstate Commerce


City of Philadelphia v. New Jersey (1978) pg. 268
F: NJ statute prohibiting the importation of most solid or liquid waste from outside the
state. NJSC said the law is valid and promotes health and welfare. Several New Jersey
operators and out of state users of the landfill sites sued to have the statute invalidated on
the ground that it discriminated against interstate commerce.
I: Whether a state may enact bans on articles of commerce absent legitimate public
welfare concerns.
H: No.
R: Just as Congress has power to regulate interstate movement, States are not free from
Constitutional scrutiny when they restrict that movement. When Congress does not
exercise its regulatory powers under the CC, the States may regulate activities that are
local in character subject to the restraints of the CC as laid out in case law. When a state
law is passed purely because of economic protectionism, it is invalid. If however the state
law addresses legitimate local concerns and the effects on interstate commerce are
incidental, then it will be upheld unless the burden imposed on commerce is excessive.
The State argues that the statute was designed to protect the environment and not the
economy, and that the benefits outweigh the slight burden on interstate commerce. The
City of Philly argues that this is just a legislative effort to suppress competition and
stabilize the cost of solid waste disposal for NJ residents and that the statute was
motivated by financial concerns.

CON LAW I Outline Page 60


This Court held that the NJ law is invalid because it violates the principle of
nondiscrimination by discriminating against articles of commerce from outside the
state. The NJ law isolated the state from the national economy. A state may not
accord its own inhabitants a preferred right of access over consumers in other states to
natural resources located in its borders. The Court distinguished quarantine laws, which
had often been upheld. The Court reasoned that such quarantine laws banned the
importation of materials that at the moment of transportation were hazardous. Here, by
contrast, the solid waste whose importation was prohibited by NJ endangered health only
when buried in landfill sites, by which time there was no valid reason to differentiate
between out of state and domestic garbage.
ROL: A State is without power to prevent privately owned articles of trade from being
shipped and sold in interstate commerce on the ground that they are required to satisfy
local demands or because they are needed by the people of the state.
Dissent:
Rehnquist makes an argument that the ban on NJ would promote commerce over the
health of its citizens. He saw no reason why NJ may ban the importation of items whose
movements risks contagion, but cannot ban the importation of items which, although they
may be transported into the state without undue hazard, will simply pile up in an ever
increasing danger to the public health and safety. Just because NJ must dispose of its
own waste does not mean that it must serve as a depository for every other state.
Class Notes:
Discrimination undermines the idea of national unity and free trade. On the issue of
intent to discriminate or not how do you prove? And even if you do so what! It
doesnt hurt free trade any less even if you can prove that the statute was not passed with
discriminatory intent. You look at the discriminatory effect and not the intent. Perhaps
this argument could have been avoided if the Governor of NY got Congress to enact a
statute saying that the States may deal with garbage as they wish. On the other hand
Philly may have gotten Congress to preempt the regulation of the states in this area.
Should Congress be able to intervene in this way? Does the Constitution allow the States
to burden other states in terms of interstate commerce? If there were a statute that was
unclear, the SC would probably either nullify the statute or find against the state being
able to regulate interstate commerce. The Sc held that this law was unconstitutional
because it concluded that it was NOT the least restrictive alternative, and that NJ could
have attacked this problem by regulating all in and out of state waste.

C&A Carbone, Inc v. Clarkstown (1994) pg. 275


F: NY enacted a flow control ordinance. The practical effect of the ordinance was to
require that any trash generated in the town be taken to a particular waste transfer
station, which charged a tonnage fee for all trash it processed.
I: Whether a law that deprives out-of-state businesses access to local markets is invalid as
discriminatory under the Commerce Clause.
H: Yes, reversed.
R: The rationale for the rule against discrimination is to prohibit local laws whose object
is local economic protectionism. The CC invalidates local laws that impose commercial
barriers or discriminate against an article of commerce by reason of its origin or
destination out of state. This ordinance falls under this category and is no less

CON LAW I Outline Page 61


discriminatory because in-state processors are also covered by the prohibition. The
ordinance hoards solid waste, and the demand to get rid of it, for the benefit of the
preferred processing facility. Discrimination against interstate commerce in favor of
local business or investment is per se invalid, except where it can be shown that there is
no other way to advance a legitimate local interest.
As a discriminatory protectionist statute, the state must now demonstrate that:
(1) the statute at issue has a legitimate purpose and (2) there are no alternative
nondiscriminatory means available. Here the Court found nondiscriminatory alternatives:
first, "the most obvious would be uniform safety regulations enacted without the object to
discriminate. These regulations would ensure competitors like Carbone do not under
price the market by cutting corners on environmental safety. . ."
ROL: State and local governments may not use their regulatory power to favor local
enterprise by prohibiting patronage of out of state competitors or their facilities.
Concurrence:
The ordinance does not discriminate against interstate commerce because in state
competitors are equally burdened. However the ordinance is an excessive burden on
interstate trade when considered in relation to the local benefits conferred. The proper
analysis is benefit versus burden, not discrimination.
Dissent:
The town here is regulating a traditional government function, something it may
legitimately do
Class Notes:
The discrimination here is that out of state trash disposing companies cannot bid for the
business. Extra costs are also imposed on outsiders and they end up financing the
Clarkstown facility. The court said that there was a less restrictive alternative of taxing
and using the subsidy (financing it with taxes and bonds) that would accomplish the same
end without involving that burden on interstate commerce. How can we tell if this
ordinance is discriminatory? It does not distinguish between in and out of state. However
discrimination against IC in favor of local businesses is per se invalid (unless there are no
other less restrictive means and there were in this case).

*Maine v. Taylor (1986) pg. 282 (upholding a statute that is facially discriminatory)
Facts: The Court upheld the constitutionality of a Maine statute that prohibited the
importation of live baitfish. Justice Blackmun noted that the statute affirmatively
discriminated against interstate transactions and therefore could be upheld only if it
survived the strictest scrutiny. The burden was on the state to show that the statute
served (1) a legitimate local purpose (2) and that the purpose could not be served as well
by an available nondiscriminatory means. Here, the Court held, that, in light of the trial
courts finding of fact, both branches of the test had been satisfied. Although it was not
certain that importation of the baitfish adversely affect the environment, Maine has a
legitimate interest in guarding against imperfectly understood environmental risks,
despite the possibility that they may ultimately prove to be negligible. Moreover, the
mere abstract possibility of developing an acceptable testing procedure did not
constitute an available nondiscriminatory alternative. Nor was there convincing evidence
that the statute was the product of a protectionist intent. The states justification was not
undermined by the fact that other states my not have enacted similar bans because

CON LAW I Outline Page 62


Maines fisheries are unique and unusually fragile. Nor was it relevant that fish can
swim directly into Maine from New Hampshire because impediments to complete
success cannot be grounds for preventing a state from using its best efforts to limit an
environmental risk.
*If a statute is facially discriminatory the burden shifts to the states to overcome two
obstacles:
*First, there must be a legitimate purpose for enacting the
statute.
*Second, there must be no alternative nondiscriminatory means available
Class Notes:
The test is whether there is a legitimate purpose, and if there is a reasonable alternative
other than the discriminatory approach (least restrictive alternative).

Co. v. Madison (1951)_pg. 282 (Reasonable alternatives and the relevance of


harms to in-state producers)
Facts: The Court invalidated an ordinance adopted by Madison City Council that
prohibited the sale of milk in the city unless it had been bottled at an approved plant
within five miles of the city. The "avowed purpose" ensuring by inspection of bottling
plants that the milk was bottled under sanitary conditions, was acceptable. But the
"practical effect" was to prevent the sale in Madison of wholesome milk produced in
Illinois and in parts of Wisconsin. "In thus erecting an economic barrier protecting a
major local industry against competition from outside, Madison plainly discriminates
against interstate commerce. This it can not do, even in the exercise of its unquestioned
power to protect the health and safety of its people, if reasonable nondiscriminatory
alternatives, adequate to serve local interest, are available." The Court concluded that
two alternatives existed: First, Madison could send its inspectors to Illinois and charge
the reasonable costs of inspection to the importing producers or; second, it could rely on
inspections by federal authorities complying with the regulatory standards in Madison
ordinance which itself adopted the provisions of a model ordinance recommended. "To
permit Madison to adopt a regulation not essential for the protection of local health
interests and placing a discriminatory burden on interstate commerce would invite
multiplication of preferential trade areas destructive of the very purpose of the Commerce
Clause."
See footnote "It is immaterial that Wisconsin milk from outside Madison area is subjected
to the same proscription as that moving in interstate commerce."

West Lynn Creamery, Inc. v. Healy (1994) pg. 284


Facts: Massachusetts taxed all milk sales in the state. The taxes, collected on sales of
milk produced both in the state and outside it, went to a subsidy fund whose proceeds
were then distributed to Massachusetts milk producers. An out of state producer
challenged the combined nondiscriminatory tax-plus-local-subsidy program. The state's
principal argument is that because "the milk order achieves its goals through lawful
means, the order as a whole is constitutional."
Holding: A pure subsidy funded from a general revenue ordinarily imposes no burden on
interstate commerce, but merely assists local business. The pricing order in this case,
however, is funded principally from taxes on the sale of milk produced in other states.

CON LAW I Outline Page 63


The Court further reasoned that "Massachusetts has created a program more dangerous to
interstate commerce than either part alone. Nondiscriminatory measures, like the
evenhanded tax at issue here, are generally upheld, in spite of adverse effects on interstate
commerce, in part because the existence of a major in-state interest adversely affected is a
powerful safeguard against legislative abuse. However, when a nondiscriminatory tax is
coupled with a subsidy to one of the groups hurt by the tax, a state's political process can
no longer be relied upon to prevent legislative abuse.
Concurrence Scalia "The wisdom of a messianic insistence on a grim sink-or-swim policy
of Laissez-faire economics would be debatable had Congress chosen to enact it; but
Congress has done nothing of the kind. It is the Court which has imposed the policy
under the dormant Commerce Clause, a policy which bodes ill for the values of
federalism which have long animated our constitutional jurisprudence."
Notes *The politics of subsidies. Why are subsidies from general revenues permissible,
but ones from industry-specific taxes impermissible?
Class Notes:
We do not want all of the benefits to fall in the state and all of the costs to fall outside. If
the state had not imposed a special tax on milk but instead paid the subsidy out of general
state revenues, it would not have been against IC. When the state exercises its taxing
power, it is not a market participant.
If its facially discrim, you use strict scrutiny. If neautral, you use balancing
approach.
In Camps Newfound Inc. v. Town of Harrison the Court held unconstitutional a tax
statute that exempted property owned by local charitable organizations but denied the
exemption to organizations operated principally for non-residents. The Court first
concluded that the statute would violate the Commerce Clause if it were targeted at
profit-making organizations because it encouraged them to "limit their out-of-state
clientele" in a way that would lead to economic balkanization. It was functionally an
export tariff that targeted out of state customers by taxing the businesses that principally
serve them.

Exceptions to the Prohibition of Facial Discrimination against IC

The Market-Participant Doctrine A State may discriminate when it acts as a


participant in the market rather than a regulator. This is sometimes an exception to strict
scrutiny, if they can prove that they are a valid market participant, and that the statute at
hand affects the market they claim to be members of.

Market Participation versus Regulation


* Hughes v. Alexandria Scrap Corp pg. 289
(SC held this was market participation and upheld)
Facts: Maryland maintained a program designed to reduce the number of junked
automobiles within the state. Maryland program, state run enterprise, was designed to
purchase scrap. A "bounty" was established on Maryland-licensed junk cars, and the state
imposed more stringent documentation requirements on out-of-state scrap processors
than on in-state ones. The Court rejected a Commerce Clause attack on the program,

CON LAW I Outline Page 64


although it noted that under traditional Commerce Clause analysis the program might be
invalid because it had the effect of reducing the flow of goods in interstate commerce.
The Court concluded that Maryland's action was not "the kind of action with which the
Commerce Clause is concerned, because, "nothing in the purpose animating the
Commerce Clause prohibits a state, in the absence of congressional action, from
participating in the market and exercising the right to favor its own citizens over others."

Reeves, Inc. v. Stake pg. 289


(SC held this was market participation and upheld)
The Court upheld a South Dakota policy of restricting the sale of cement from a state-
owned plant to state residents only, declaring that "the basic distinction drawn in
Alexandria Scrap between states as market participants and states as market regulators
make good sense and sound law." The Court relied upon "the long recognized right of
trader of manufacturer, engaged in an entirely private business freely to exercise his own
independent discretion as to parties with whom he will deal." In essence the Court
recognized the principle that the Commerce Clause places no limitations on a state's
refusal to deal with particular parties when it is participating in the interstate market in
goods.
Class Notes:
Why was this not considered discriminatory? The cement factories were privately owned.
If it were the state owned factories passing such a law, under NY v. Philly it would
probably be struck down and it would have to survive strict scrutiny. This is a federalism
issue and the Court probably said the law was ok since this was done by as a state policy
and the Court recognized that the state had the sovereignty to carry out this policy.

White v. Massachusetts Council of Construction Employers


(SC held this was market participation and upheld)
The Court sustained, against a Commerce Clause challenge, an executive order of the
Mayor of Boston that required all construction projects funded in whole or part by city
funds or city administered funds to be performed by work force of at least 50% city
residents. The Court rejected the argument that the city was not entitled to the protection
because the order had the effect of regulating employment contracts between public
contractors and their employees. The fact that the employees were "working for the city"
was crucial to the market participant.

South-Central Timber Development v. Wunnicke pg. 289


(SC held this was regulation and was unconstitutional)
Facts: Alaska proposed to sell 49 million board feet of timber it owned. Its proposed
contract required the successful bidder to process the timber before it was shipped out of
Alaska. The stated purpose was to protect existing timber processing industries, promote
new industries, and derive revenue from all timber resources. South-Central is an Alaska
corporation that purchases and logs timber and then ships it elsewhere primarily to Japan,
where it is processed. It sought an injunction against the local processing requirement as
a violation of the Commerce Clause. The court of appeals denied the injunction, finding
implicit congressional authorization for the requirement. The Supreme Court did not find

CON LAW I Outline Page 65


any congressional authorization stating "congressional consent must be unmistakably
clear.
Issue: Whether Alaska acting as a market participant may require buyers of raw lumber to
process that lumber in-state through privately owned processing company.
No. First, Alaska argued first that the its primary manufacture requirements fits squarely
within the market-participant doctrine, arguing that "Alaska entry into the market may be
viewed as precisely the same type of subsidy to local interests that the Court found
unobjectionable in Alexandria Scrap." However, when Maryland became involved in the
scrap market it was a purchaser of scrap; Alaska, on the other hand, participates in the
timber market, but imposes conditions downstream in the timber-processing market.
Second, Alaska also found Reeves controlling "Reeves made it clear that the Commerce
Clause imposes no limitation on Alaska's power to choose the terms on which it will sell
its timber." Although the Court in Reeves did strongly support the right of a state to deal
with whomever it chooses when it participates in the market, it did not and does not
purport to sanction the imposition of any terms that the state may desire. Finally, Alaska
argues that since the Court in White upheld a requirement that reached beyond "the
boundary of formal privity of contract then a fortiori, the primary manufacture
requirement is permissible, because the state is not regulating contracts for resale of
timber or regulating the buying and selling of timber, but instead "a seller of timber, pure
and simple. Yet, it is clear that the state is more than merely a seller of timber. In the
commercial context the seller usually has no say over and no interest in how the product
is to be used after sale; in this case, however, payment for the timber does not end the
obligations of the purchaser. The limit of the market participant doctrine must be that it
allows a state to impose burdens on commerce within the market in which it is a
participant, but allows it to go no further.

Distinguishing regulation from market participation:


The problem in the regulatory timber case is that the restriction went beyond the
transaction itself (purchase of timber) and restricted post-purchase activity (the
processing of timber out of state). The State may not impose conditions that have a
substantial regulatory effect outside of that particular market. Downstream restrictions
have a greater regulatory effect than do limitations on the immediate transaction.
The Market Participant Doctrine does not completely exempt state activities from
compliance with the CC, instead it exempts them in the absence of a congressional
prohibition.

The Privileges and Immunities Clause Limitation on the Market


Participant Exception
Art IV, cl. 2 "The citizens in the several States shall be entitled to all
the Privileges and Immunities of Citizens in the several States."
The purpose of this clause is to promote interstate harmony. The clause does not preclude
discriminating against citizens of other states where there is a substantial reason for the
difference in treatment.
United Building & Construction Trades Council v. Camden (1984) pg. 292
Facts: Camden passed an ordinance requiring that at least 40% of the employees of
contractors and subcontractors on city projects be Camden residents in pursuant to a

CON LAW I Outline Page 66


statute authorizing cities to adopt affirmative actions plans. The council challenged the
ordinance as a violation of the privileges and immunities clause. The N.J Supreme Court
upheld the ordinance.
Issue: Does the Privileges and Immunities Clause apply to municipal ordinances which
discriminate on the basis of municipal rather than state citizenship?
Holding: Yes, given the Camden ordinance, an out of state citizen who ventures into N.J
will not enjoy the same privileges as the N.J citizen residing in Camden. It is true that
N.J citizens not residing in Camden will be affected by the ordinance as well as out of
state citizens.
Distinguishing White: Our decision in White turned on a distinction between the city
acting as a market participant and the city acting as a market regulator. The question
whether employees of contractors and subcontractors was crucial to that analysis. The
question had to be answered in order to chart the boundaries of the distinction. But the
distinction between market participant and market regulator relied on in White to dispose
of the Commerce Clause challenge is not dispositive in this context. The two clauses have
different aims and set different standards for state conduct.
Application of the Privileges and Immunities Clause to a particular instance of
discrimination against out of state residents entails a two-step inquiry.
1. As an initial matter, the court must decide whether the ordinance burdens one of
those privileges and immunities protected by the Clause. That is a fundamental
right. Here employment by a state the Court found not to be considered a
fundamental right.
2. Then if there is a "substantial reason" for the difference in treatment it may be
justified.
A state may discriminate in its hiring practices: For the purpose of distinguishing White
from Camden the Court held that the market participant doctrine barred a claim against a
city for requiring contractors to hire a certain percentage of city residence. In White the
Court reasoned that although the contractors were independently hired, they were
nevertheless employed by the city. In Camden, on the other hand, the Court in applying
the Privileges and Immunities Clause found that once a city hired a contractor, the city
was not the employer. The Camden Court held that a state may discriminate in its hiring
practices because state employment is not a fundamental right that invokes protection
from the Privileges and Immunities Clause. See Notes from Schnably
Class Notes:
This comes up when dealing with local residents. Even if something is ok under the
Market Participant Clause, you still have to worry about the Privileges and Immunity
Clause however the PI Clause only applies to individuals and not when you
discriminate against corporations.
Does the Market Participant Doctrine change if the state owns the business? The SC
respects the states to say that they do have the right to regulate as long as it does not
extend past the transaction (Alaska timber)

The Commerce Clause

CON LAW I Outline Page 67


What does Art IV sec. 2 add to the Commerce Clause? In Toomer v. Witsell the Court
invoked Art. IV to hold unconstitutional a South Carolina statute imposing a $2,500
license fee on nonresidents commercial shrimp fisherman, in contrast to the $25 fee for
residents. In many ways, the modern function of Art IV sec. 2 appears to be that of
carving out the exception to the market participant exception to the Commerce Clause.
Note, however, that corporations are not "citizens" within the meaning of the Privileges
and Immunities Clause.

Equal Protection Clause


The EPC allows discrimination if it is a rational means to accomplish a legitimate state
purpose. Under the CC, the states interest is weighed against the burden imposed on
interstate commerce, whereas under the EPC, as long as the state interest is legitimate the
state law stands if the burden imposed is rationally related to that purpose.

Facially Neutral Statutes

Exxon Corp. v. Governor of Maryland (1978) pg. 295


(held that the Maryland statute was constitutional)
Facts: Maryland passed a law prohibiting oil producers or refiners from operating retail
gas stations in Maryland. The law was enacted because there was evidence that gas
stations, operated by producers and refiners, had received preferential treatment during
the 1973 oil shortage. Since no gas is produced or refined in Maryland, the rule against
vertically integrated operations affected out-of-state companies exclusively. Conversely,
the vast majority (but not all) of the non-integrated retailers, who were not harmed and
were probably helped by the statute, were in-state business people. Exxon and several
others out-of-state integrated oil companies sued. They made a three pronged Commerce
Clause argument: (1) that the measure impermissibly discriminated against interstate
commerce; (2) that the measure unduly burdened such commerce; and (3) that because of
the nationwide nature of oil marketing, only the federal government may regulated retail
gas.
Holding: The Court upheld the statute against each of these attacks. First, the statute did
not discriminate against interstate commerce. Plainly, the Maryland statute does not
discriminate against interstate goods, nor does it favor local producers and refiners.
Since there are no local producers or refiners such disparate treatment between interstate
and local commerce would be meritless. Most significantly, not all out-of-state-
companies were affected by the statute; Sears Roebuck, for instance, was an out-of-state
company which was selling gas at retail within Maryland, yet was not involved in
refining it and was therefore not affected by the statute. The mere fact that the entire
burden of the statute fell on some out-of-state companies was insufficient to establish that
interstate commerce was discriminated against. Similarly, the Court found that
interstate commerce was not impermissibly burdened by the statute. The majority
conceded that the statute might cause sales volume to shift from refiner operated stations
to independent dealers. But, the Court held, the Commerce Clause protects the interstate
market, not particular interstate firms, from prohibitive or burdensome regulations.
Furthermore, the Court noted, in all probability the same percentage of gasoline would

CON LAW I Outline Page 68


come from out-of-state suppliers after the statute as before it. Finally, the Court quickly
dismissed the contention that because the market for gasoline is nationwide, no state may
regulate its retail marketing. The Dormant Commerce Clause may preempt an entire
field from state regulation only when lack of national uniformity would impede the flow
of interstate goods. See Cooley What the Plaintiffs were complaining of here was not a
lack of uniformity, but rather that many or all of the states would pass exactly the sort of
divestiture law that Maryland did. Thus the problem was not one of national uniformity.
Dissent: } Blackmum
The Commerce Clause forbids discrimination against interstate commerce, which
repeatedly has been held to mean that states and localities may not discriminate against
the transactions of out-of-state actors in interstate markets. See Dean Milk
Discrimination need not appear on the face of the state or local regulation. The
Commerce Clause forbids discrimination, whether forthright or ingenious. In each case
it is our duty to determine whether the statute under attack, whatever its name may be,
will in its practical operation work discrimination against interstate commerce. The
state argues that discrimination against interstate commerce has not occurred because
No nexus between interstate as opposed to local interest inheres in the production or
refining of petroleum. Although this statement might be correct in the abstract, it is
incorrect in reality, given the structure of the Maryland petroleum market.
The CC protects the interstate market, not particular interstate firms, from prohibitive or
burdensome regulations. A nondiscriminatory regulation serving substantial state
purposes is not invalid simply because it causes some business to shift from a
predominantly out of state industry to instate. Only if the burden on IC clearly outweighs
the States legitimate purposes does such a regulation violate the CC.
Class Notes:
Argument that it is not discriminatory: The law does not distinguish between in and out
of state entities.
Argument it is discriminatory: the effect of the law only adversely impacted out of state
companies.
We have to look at the activity involved (here retailing gasoline) and see if the law
prevents outsiders from performing activity. The Court here says that the law does not
shut out outsiders from owning and running a gas station.
The state may argue that this does not affect commerce because the same amount of
gasoline will be sold.
Reconciling with Hunt: Hunt specifically discriminates against out of state and gives
itself an advantage. The majority says that there is no impact on commerce period.

Hunt v. Washington State Apple Advertising Commission (1977) pg. 256


(Held the regulation unconstitutional)
Facts: North Carolina enacted a statute requiring all closed containers of apples sold or
shipped into the state to bear no grade other than the applicable U.S. grade or standard.
Washington, the countrys largest producer of apples, requires that apples be tested and
graded under a system of grades superior to the standards adopted by the United States
Department of Agriculture.
Issue: Whether North Carolina statute facially discriminates against interstate commerce
in that it requires Washington apple producers to adhere to a lower standard. And if it is

CON LAW I Outline Page 69


facially discriminatory can the state overcome the burden by showing that the statute
effectuates a legitimate public purpose and that the means that the state has chosen the
least discriminatory.
Holding: The Challenged statute has the practical effect of not only burdening interstate
sales of Washington apples, but also discriminating against them.
1. The first is the statutes consequence of raising costs of doing business in North
Carolina market for Washington apple growers and dealers, while leaving those of
their North Carolina counterparts unaffected.
2. Second, the statute has the effect of stripping away from the Washington apple
industry the competitive economic advantage it has earned for itself through its
expensive inspection and grading system.
3. Third by prohibiting Washington growers and dealers from marketing apples
under their States grades, the statute has a leveling effect which insidiously
operates to the advantage of local apple producers. The Washington State grades
are equal or superior to the USDA grades in all corresponding categories. Hence,
the North Carolina statute is facially discriminatory (even though NC argued that
its for consumer protection) (but the problem w this is that no label is required)
(plus Washington State label is NOT better than USDA label so how could that
serve consumer interests?) (plus consumers never see it b/c its the label on the
box, not the individual apples in the store). So how is this consumer protection?
(well you could argue that retailers see it and advertise about it) (there were less
restrictive alternatives).prof says that this case, even though in facially neutral
section, is not facially neutralb/c the court seems to be applying a pretty strict
scrutiny and they treat it as if its facially discrim, as opposed to first treating it as
its neutral and then later applying strict scrutiny.
When discrimination against commerce of the type we have found is demonstrated, the
burdens falls on the state to justify it both in terms of the local benefits flowing from the
statute and the unavailability of nondiscriminatory alternatives adequate to preserve the
local interest. The Court found that the statute design to guard against fraud does
remarkably little to further that laudable goal. Secondly, it appears that
nondiscriminatory alternatives to the outright ban of Washington State grades are readily
available.
In addition, the Court found that this was NOT the least restrictive alternative. For
example, NC could have used the USDA label as a benchmark and allowed for out of
state grading.
Class Notes:
The majority looks more at the discriminatory effect as opposed to the intent. No grade
had to be put on the stamp at allthe standard just didnt permit any other grade except
for that states.

South Carolina Highway Department v. Barnwell Brothers (1938) pg. 307


The Court upheld the constitutionality of a South Carolina statute prohibiting the use on
state highways of trucks that were over ninety inches wide or over 20,000 pounds in
gross weight. The Court stated The commerce clause by its own force prohibits
discrimination even when state legislation nominally of local concern is in point of fact
aimed at interstate commerce, or by its necessary operations is a means of gaining a local

CON LAW I Outline Page 70


benefit. . . the test is whether the state legislature in adopting regulations has acted within
its province, and whether the means of regulation chosen are reasonably adapted to the
end sought. Such regulation in the absence of Congressional action has for the most part
been left to the states.

So. Pacific Co. v. Arizona (1945) pg. 307


Court held unconstitutional a 1912 statute limiting train lengths to 14 passengers and 70
freight cars. Because of the need for national unity the states may not regulate since it
should come from a single authority. Balance of local safety interests versus the effect on
national uniformity. This is distinguished from highways, which are local in nature. The
state interest is outweighed by the national interest in an adequate, economical and
efficient railway transportation service, which must prevail.

Note *when a facially non-discriminatory statute is involved purporting to regulate


interstate commerce the court applies a balancing test. Namely whether the regulation's
purpose and resulting effects when weighed against the interference with interstate
commerce does not disproportionately interfere with interstate commerce.

Kassel v. Consolidated Freightways Corp. (1981) pg. 265


(Held the Iowa statute was unconstitutional because the burden on IC was greater than
the safety benefits).
Facts: Consolidated, one of the nation's largest common carriers, used two types of
trucks: the semi 55 feet long, and the double a 65-foot long combination of a tractor and
two trailers. It operated on Interstate Routes 80 & 35, major transportation corridors
through Iowa. Unlike all other states in the West and Midwest, Iowa generally prohibits
the use of 65-foot doubles within its borders. Instead, most truck combinations are
restricted to 55 foot in length. Doubles, mobiles homes, trucks carrying vehicles such as
tractors and other farm equipment, and singles hauling livestock, are permitted to be as
long as 60 feet. Notwithstanding these restrictions, Iowa's statute permits cities abutting
the state line by local ordinance to adopt the length limitations of the adjoining state.
Where a city has exercised this option, otherwise trucks are permitted within the city
limits and in nearby commercial zones. Because of Iowa's statutory scheme, consolidated
cannot use its 65-foot doubles to move commodities through the state. Instead the
company must do one of four things (1) use 55 foot singles; (2) use 60 foot doubles; (3)
detach the trailers of a 65 foot double and shuttle each through the state separately; or (4)
divert 65 foot doubles around Iowa. In a 14 day trial, both sides adduced evidence on
safety, and on the burden on interstate commerce imposed by Iowa's law. On the question
of safety, the District Court found that the "evidence clearly establishes that the twin is as
safe as the semi."
Issue: Whether Iowa's statutory scheme requiring only certain type of trucks on their
highway to promote safety interferes with interstate commerce so substantially that the
statute is an unconstitutional.
Yes. The incantation of a purpose to promote public health or safety does not insulate a
state law from a Commerce Clause attack. Regulations designed for the salutary purpose
nevertheless may further the purpose so "marginally" and interfere with interstate

CON LAW I Outline Page 71


commerce so "substantially" as to be invalid under the Commerce Clause. Here the state
failed to present any persuasive evidence that 65-foot doubles are less safe than 55-foot
singles. The Court normally does accord "special deference" to state highway safety
regulations. This traditional deference derives in part from the assumption that where
such regulations do not discriminate on their face against interstate commerce, their
burden usually falls on the challenging party. Less deference to the legislative judgment
is due, however, where the local regulations bears disproportionately on out-of-state
residents and businesses. Such a disproportionate burden is apparent here. Iowa's
scheme, although generally banning large doubles from the state, nevertheless has several
exemptions that secure Iowans many of the benefits of large trucks i.e. allowing cities to
adopt ordinances near the state border would, in effect, allow those cities to reap the
benefit of goods entering into the state while keeping the cost of intrastate highway repair
to a minimum while shunting to neighboring states many of the cost associated with their
use. Because Iowa has imposed this burden without any significant countervailing safety
interest, its statute, violates the Commerce Clause.
Concurrence: The SC does not have to decide whether in fact the regulation promotes its
intended purpose (safety). It is clear that the Iowa law sought to discourage traffic. Thus,
the safety advantages and disadvantages of the types of trucks are irrelevant to this
decision. The traffic concern is protectionist in nature and impermissible under the CC.
This attempt should be subject to a virtually per se rule of invalidity.
Dissent Rehnquist (According to Schnably this is a good example of legislative intent,
dissent didnt think it was motivated by protectionism)
Iowa's action in limiting the length of trucks which may travel on its highway is in no
sense "unusual." Every state in the Union regulates the length of vehicles permitted to
use the public roads. A determination that a state law is a rational safety measure does
not end the Commerce Clause inquiry. A "sensitive consideration of the safety purpose in
relation to the burden on commerce is required to determine whether the asserted
justification is merely a pretext for discrimination against IC. When engaging in such a
consideration the Court does not directly compare safety benefits to commerce cost and
strike down the legislation if the latter can be said in some vague sense to "outweigh" the
former. In reviewing a state highway regulation where Congress has not acted, a court is
not called upon, as a state legislatures, to determine what, in its judgment, is the must
suitable restriction to be applied of those that are possible, or to choose that one which in
its opinion is best adapted to all the diverse interests affected. "The question of safety in
the circumstances of this case is essentially a matter of public policy, and public policy
can, under our constitutional system, be fixed only by the people acting through their
elected representatives." The question here is rather whether it can be said that the
benefits flowing from Iowa from a rational truck length limitation are "slight or
problematic." The particular line chosen by Iowa e.g. 60 feet is relevant only to the
question whether the limit is a rational one. Once a court determines that it is, it
considers the overall safety benefits from the regulation against burdens on interstate
commerce, and not any marginal benefits from the scheme the state established as
opposed to that the plaintiffs desire. Furthermore, the effort in both the plurality and
concurring opinions to portray the legislation involved here as "protectionist" is in
error. Whenever a state enacts more stringent safety than its neighbors, in an area that
affects interstate commerce, the safety law will have the incidental effect of deflecting

CON LAW I Outline Page 72


interstate commerce to the neighboring states. Indeed, the safety and protectionist
motives cannot be separated: the whole purpose of safety regulation of vehicles is to
protect the state from unsafe vehicles. If neighboring state chooses not to protect not to
protect its citizens from the danger discerned by the enacting state, that is its business, but
the enacting state should not be penalized when the vehicles it considers unsafe travel
through the neighboring state.

II. Dissent (Rehnquist) standard in balancing is more deferential. His is not safety
benefits against costs but if his standard were the prevailing one, youd have to show that
the benefits from the regulation are DEMONSTRABLY TRIVIAL; and that he
BURDEN ON CONGRESS IS GREAT. (trivial benefits and really huge costs)
Class Notes:
The plurality does not adopt Brennans approach of trying to look at what the legislators
intent was. The plurality says the statute was not facially discriminatory, however it does
not pass muster under the balancing test. The plurality did not buy the argument about the
safety benefits so the balancing test was easy since the costs in terms of burden on IC
was clear.
Inherently National or Local.
The facts really count here in terms of understanding legislative intent.
Balancing was easy here because the court felt there was no significant safety benefit to
the state law. SO THE DISTRICT COURT WAS HIGHLY FACT INTENSIVE/decisive
in actually determining that the bigger trucks were no less safe.

Cost/benefit analysis. The cost for the bigger trucks were 12MILL/year more and the
extra benefit was NIL.
Rehnquist (dissent):
Hes saying that its true that 60 doesnt make it safer than 55. And that the benefits of
where you draw a line are trivial, HOWEVER thatsnot important: in general the most
important thing is analyzing the benefits of a length limitationhe puts his finger on the
scale better than the majorities b/c he wants to GIVE DEFERENCE OT THE STATES.
National regulation does not necessarily mean national uniformity. They could have
delegated the regulation to an administrative agency.

Doctrinal Framework:
You want to first look at whether the law is facially discriminatory if not you do the
balancing test, if yes you apply strict scrutiny. The difficult part is figuring out whether it
is facially discriminatory.

Obstructing commerce. In Buck v. Kuykendall (1925) pg. 320, Buck, a citizen of


Washington, wished to operate an auto stage line between Portland, Oregon and
Seattle, Washington. After obtaining a license from Oregon, he applied to the
Washington authorities for a certificate that would allow him to begin operations. The
certificate was denied on the grounds that the route was already adequately served by
railroads and four other auto stage lines. The Court held that the denial violated the
Commerce Clause. The purpose was the prohibition of competition. Its effect was
not merely to burden interstate commerce, but to obstruct it. The Court also noted that

CON LAW I Outline Page 73


the prohibition defeated the purpose of Congress in legislation providing federal aid for
highway safety.

Actual Purpose Review


Focus on action as well as inaction. It is difficult to get at what the actual intent was.

Preemption
pp. 324-328
Congress may preempt state regulation. The question whether federal law preempts a
certain state action is one of congressional intent.

*There are three ways in which Congress may preempt state regulation:
1) Express Preemption - by expressly stating the preemption
2) Field Preemption - by enacting a scheme/system of regulations so
comprehensive as to displace all state regulations even if they do not
conflict with any specific federal one "occupying the field"
3) Conflict Preemption - by enacting a regulation with which the state
regulation in fact conflicts. It is impossible to comply with state and
federal.

Gade v. National Solid Waste Management Association (1985) pg. 325


(held that preemption applies where a state statute is not consistent with the purpose and
structure of a federal statute as a whole.)
Facts: In 1988 Illinois enacted two statutes for licensing workers who handle hazardous
waste. The statute had the stated purpose of "promoting job safety" and "protecting life,
limb, and property." They require a license applicant to provide a certified record of at
least 40 hours of training under an approved program in Illinois, to pass a written
examination, and to complete an annual refresher course. The federal Occupational
Safety and Health Administration (OSHA) promulgated regulations on training for
hazardous waste workers. These regulations, authorized by (OSHA) require workers
receive a minimum of 40 hours of instruction off the site where they may be exposed to
hazardous wastes and a minimum of three days of field experience under the supervision
of a trained supervisor.
Issue: Whether the Illinois statute mandating certain requirements that, in effect,
outweigh the federal (OSHA) regulations is pre-empted by the federal statute.
Yes. The issue whether a certain state action is pre-empted by federal law is one of
congressional intent. "The purpose of Congress is the ultimate touchstone." Preemption
may be either expressed or implied and
"is compelled whether Congress' command is explicitly stated in the statute's language or
impliedly contained in its structure and purpose." Absent explicit pre-emptive language
there are two types of implied preemption: first field preemption, where the scheme of
federal regulation is "so persuasive as to make reasonable the inference that Congress left
no room for the states to supplement it." See Rice v. Santa Fe Elevator Corp. And
second, conflict preemption where "compliance with both federal and state regulation is
a physical impossibility, or state law "stands as an obstacle to the accomplishment and

CON LAW I Outline Page 74


execution of the full purpose and objectives of Congress. Here the Illinois statute is in
direct conflict with section 18(a) of OSHA. Section 18(a) of OSHA expressly provides
that a state may remove itself from the OSHA regulations entirely by submitting its own
plan and having it approved by the Secretary of Labor. The Court did not interpret
section 18(a) as allowing a state to merely add to and not entirely remove itself from, the
OSHA rules without seeking approval because such an interpretation would make section
18(a) superfluous. Further, such an interpretation would contravene Congress' intent to
subject employers and employees to only one set of regulations, whether federal or state.
And despite the fact that the state can demonstrate some purpose other than that
underlying the OSHA regulations for its acts, the acts are nonetheless preempted because
they are directed at workplace safety.
Dissent Souter, J
The language of OSHA is insufficient to demonstrate any intent to
Preempt state law as found by the Court. The law presumes Congress did not intend to
displace state law, and where the field is one traditionally occupied by the state (such as
the police power involved here) there is no preemption unless that is the clear and
manifest intent of Congress to preempt.
Class Notes:
First we would look to see if whether there was express preemption. There may be some
provision that only the feds can regulate. For express preemption you have to be able to
point to something specific. There could also be conflict preemption where you would
not be able to satisfy both state and federal regulations.
McCullough v. Maryland preemption and the bank.
One take is that this is a very pro-business decision.

The Contracts and Takings Clauses


Pp 942-957
In the 1930s there was a shift in the Commerce Clause decisions. These decisions are in
line with that shift.

Government ought not to be permitted to redistribute resources by taking resources


from one person for the benefit of another. The Contracts Clause bars state government
from disrupting voluntary agreements simply because government wants to help one side
or another. The Eminent Domain Clause bars the federal government (and states after
incorporation) from taking property from one person and giving it to someone else.

Contracts Clause
No State shall pass any law impairing the obligation of Contracts Art. I Sec. 10 cl. 1.
This clause is only applicable to the states, not the federal government.
Until the BOR, this was one of the few restraints on the states in the constitution. Why
was the freedom to contract singled out by the framers? Because of debtor relief.

Prospective or retrospective

Interfering with Contracts under Police Power an exception to the Contracts Clause

CON LAW I Outline Page 75


Regulation versus Impairment

Home Building & Loan Association v. Blaisdell (1934) pg. 945


(Held that the MN statute did NOT violate the Contracts Clause)
F: During depression, Minnesota passed a mortgage moratorium law providing relief to
homeowners threatened with foreclosure. The lower court upheld the statute.
I: Whether legislation temporarily postponing enforcement of mortgages, in an
emergency situation, violates the Contracts Clause.
H: No, affirmed.
R: The Minnesota statute does not violate the Contracts Clause of the constitution.
Emergency does not increase granted power or remove or diminish the restrictions
imposed upon power granted or reserved. Conditions may arise where temporary restraint
of enforcement may be constitutional. A court will not use the CC to throttle the ability of
states to protect their fundamental interests. The reservation of a states power to exercise
its protection is read into all contracts. The states legislation aimed to protect the entire
societys basic interest, and the conditions are reasonable and temporary.
ROL: Legislation temporarily postponing enforcement of mortgages, in an emergency
situation, does NOT violate the Contracts Clause.
Dissent: The CC itself was adopted in a state of emergency with debt problems. If it was
good then it should be good now or abandon it all together. It cannot be argued that
conditions which produced the rule may now be invoked to destroy it.
Class Notes:
The majority views the constitution as one that can be adapted to the various crises of
human affairs. The majority comes close to saying we dont care what the framers
thought. In this case the majority viewed the constitution as having an evolving
interpretation. When we dealt with separation of powers we looked at the issue of intent
and also with federalism dealing with who do we mean when we talk about the framers.
This case is famous for what it says about framers intent.

Energy Reserves pg. 955


Test for violation of CC: We look at whether there is a substantial impairment of the
contractual relationship. The severity of the impairment increases the level of scrutiny to
which the legislation will be subjected.

US Trust v. NJ (1977) pg. 951


(The SC here invalidated the repeal of the covenant by the states as a violation of the
Contracts Clause).
F: A NJ statute repealed a statutory covenant made earlier with NY that limited the ability
of the Port Authority of NJ and NY to subsidize rail passenger transportation from
revenues and reserves NJ and NY later concluded that subsidization was necessary. The
SC here invalidated the repeal of the covenant.
R: Trial court concluded that repealing the statute was a valid exercise of NJs police
power. However SC said the CC limits otherwise legitimate exercises of state legislative
authority even where there is an important public interest. If a state could reduce its
financial obligation whenever it wanted to spend money for what it regarded an important
public purpose, the CC would provide no protection at all. The repeal could only stand if

CON LAW I Outline Page 76


the impairment was both reasonable and necessary to serve the admittedly important
purposes claimed by the state. Here this was not satisfied.
Dissent: The CC was meant to prohibit the state from impairing the contracts of private
parties, here the original K was made by earlier legislators.
Class Notes:
Here the state is impairing its own contract. The Court wants closer scrutiny in such a
case. The Court applies a heightened scrutiny and looks for a legitimate cause and also
considers less restrictive alternatives (similar to the Dormant Commerce Clause analysis).

Allied Structural Steel v. Spannaus (1978) pg. 953


(Held that the MN act violated the Contracts Clause)
F: Company had a pension plan for its employers, which by K allow it to amend or
terminate the plan. Minnesota passed an act which would charge employers if they
terminated a pension plan. The company brought this action claiming the act was
unconstitutional for impairing its contractual obligations.
The Court said that the CC is not dead. There must be an inquiry into how severe the
impairment is on the contractual obligation. The Court concluded that the impairment in
this case was severe. There is no showing that this severe impairment was necessary to
address the social problem. The effect of this act was very narrow (only private
companies etc.) unlike the Blaisdell case which protected a broad societal interest. This
act wanted to go into effect retroactively. If the CC means anything at all it renders
Minnesotas act unconstitutional.
Dissent:
The act does not relieve either party of their contractual obligations, it simply creates an
additional duty on the employer.
Class Notes:
The more severe the impairment, the greater the level of scrutiny.
Where the state itself is not the contracting party, courts should defer to the legislative
judgment as to the necessity and reasonableness of the contractual abrogation.
Just about any regulation is going to upset a K somewhere.

The Court in such a case does the following:


Identifies the legitimate state interests
Identify a rough relation between the interest and the measure under review

In essence the Court applies this analysis under DP, EP, and Contracts Clause.

Lochner Era
pp. 710-735
The constitutional provisions that protect private economic interests from government
interference had been defined very narrowly. Lochner created the doctrine of Economic
Substantive Due Process. Ownership of property grew and with it so did state regulation
(prices, minimum wage etc.). The Court relied on the DP Clause to invalidate state
economic regulations (The Minnesota Rate Case pg. 712).
This doctrine is one of direct protection of individual rights. The doctrine is currently
discredited.

CON LAW I Outline Page 77


Lochner Doctrine:
Worrying about federalism
Not sure where to draw the line

Lochner v. NY (1905) pg. 713


(The Court held unconstitutional a NY statute limiting the hours (weekly/daily) that a
worker may work in a bakery)
F: NY enacted a statute limiting the work hours for bakery employees.
I: Whether a statute that interferes with the freedom of employer and employee to
contract with each other violates the constitution.
H: Yes, reversed.
R: The right to purchase or sell labor is part of the liberty protected by the 14th
amendment unless there are circumstances that exclude the right.
There is no reasonable foundation to validate this statute as a health law to safeguard
public health. There must be more of a direct relation and there must be a
substantial effect. To allow regulation here would lead to a slippery slope where
anything could be justifiably regulated for some greater purpose in effect taking away
all meaning from the 14th amendment. An act would be valid, carried out by police
powers if there was a material danger to public health or the health of the employees,
here the state is interfering with the rights of the individual.
ROL: A statute that interferes with the freedom of employer and employee to contract
with each other violates the constitution
Dissent: The Courts should not be concerned with the wisdom of state legislation. NY
believes that the amount of hours worked in a bakery affects public and employee health,
it is not our place to question the wisdom involved. There is room to argue over whether
there is direct relation and substantial effect. There is a connection between health and
hours worked at bakery.
Holmes Dissent:
Whether this court approves or not, state laws may regulate life in many ways, which
equally with this interfere with the liberty to contract. Examples: Sunday laws, usury
laws, lottery prohibitions.
Class Notes:

Lochner was later considered an aberration by the Court most of the Lochner cases
were overturned, and the trend has been to allow state legislation even where there is an
economic impact on private parties.

If this were a federal statute and the Court struck it down you would argue that the Court
was striking it down under the Commerce Clause. In this case the state statute is struck
down as a violation of the individual freedom to contract and that the statute is
inconsistent with substantive due process. This is not a Contracts Clause case because
since the employees work at will the statute does not impair the establishment of the
contract. For this reason the Contracts Clause is not applicable. The CC only applies to
already existing contracts, not the impairment of future contracts from being made. There
are however exceptions to this liberty to contract for example to protect the health and

CON LAW I Outline Page 78


welfare of the public. The legislature felt that bakeries were sufficiently dangerous like
mines such that regulation should be permitted. This is a paternalistic view.
Possible aims:
1. Protect the workers this is a legitimate end.
2. Help give the workers a better deal this is an illegitimate end.

The analysis is similar to the dormant commerce clause:


Does the state law impinge on a fundamental right? Yes freedom to contract.
What is the end of the regulation? health.
Is this the least restrictive means? No, they could have regulated the facilities and
equipment etc.

Why should the Court tell the legislature which way to achieve the desired end? It would
be more appropriate for the legislature as long as there is no constitutional impairment.
There is a means/ends analysis.

Criticisms of Lochner:
Textual critique:
Is this actually a constitutional right? It is not in the constitution.

Williamson v. Lee Optical (1955) pg. 731


Upheld state statute that prohibited opticians from making glasses without prescriptions.
The test on the end is whether it is legitimate (public health) then we will assume that that
is what the legislature had in mind without inquiring about their intent. The Court will not
dictate the choice of means. Here the Court will accept any plausible end as long as it is
not irrational. Today we would not start from the assumption that we cannot tell adults
how to contract. Now we look to see if there is a conceivable purpose and then we look to
see if it is irrational to see that the statutory scheme will achieve the end.

Ferguson v. Skrupa (1963) pg. 731


Court upheld statute outlawing debt adjustment.

Summary of 2 kinds of critiques of Lochner (pg. 723)


1. Substantive critique less to do with the role of the court and more to do with the
ideas in Lochner about the appropriate role of government.
2. Institutional dont infer things that arent in there. The Court overstepped its
bounds in relation to the legislature.

Exam: Look at the general themes: what themes cut across?


Judicial review is it undemocratic? Is it always right for the Court to have the final say
on interpreting the constitution as opposed to a political branch.
Interpretation: Text intent function (structure or function).
11th amendment text doesnt count for very much.
Intetnt Braisdell (who cares about framers intent). Should intent matter?
Function what works?

CON LAW I Outline Page 79


Look for persistence of different approaches example separation of powers the
various opinions use different interpretative approaches. Standing but for vs. other
approach. Same area with different approaches.
Look for the themes.

CON LAW I Outline Page 80

You might also like